Musculoskeletal Exam

PANCE/PANRE Musculoskeletal Exam

Congratulations - you have completed Musculoskeletal 143 Questions. You scored %%SCORE%% out of %%TOTAL%%. Your performance has been rated as %%RATING%% %%FORM%%

Have Your Exam Results Emailed to You

Enter your name and email address below to have your results as well as the test questions, your answers and the correct answers delivered to your inbox.
  • mTouch Quiz Populated Fields

    mTouch Quiz will automatically populate the fields below. Feel Free to add additional fields for the Quiz Taker to complete using the "Add Fields" options to the right.
  • The name of the quiz
  • The number of correct answers. This has the same value as the %%SCORE%% Variable on the Final Screen.
  • The total number of questions. This has the same value as the %%TOTAL%% Variable on the Final Screen.
  • Correct answer percentage. This has the same value as the %%PERCENTAGE%% Variable on the Final Screen.
  • Number of wrong answers. This has the same value as the %%WRONG_ANSWERS%% Variable on the Final Screen.
  • Time allowed (Requires timer add on). This has the same value as the %%TIME_ALLOWED%% Variable on the Final Screen.
  • Time used (Requires timer add on). This has the same value as the %%TIME_USED%% Variable on the Final Screen.
Your answers are highlighted below.
Question 1
Abduction of the shoulder against resistance helps localize pain in which of the following muscles of the shoulder girdle?
A
Supraspinatus
B
Infraspinatus
Hint:
Lateral rotation against resistance tests the infraspinatus and teres minor.
C
Teres minor
Hint:
See answer for explanation.
D
Subscapularis
Hint:
Medial rotation against resistance tests the subscapularis.
Question 1 Explanation: 
Abduction against resistance tests the supraspinatus
Question 2
A 22 year-old male presents to the ED after sustaining a blow to the knee during football practice. The knee exam demonstrates significant forward translation of the tibia when the knee is in 15 degrees of flexion and external rotation at the hip. Which of the following knee maneuvers does this represent?
A
Abduction stress test
Hint:
The abduction stress test is performed to evaluate medial collateral ligament tears while applying valgus stress.
B
Anterior drawer sign
Hint:
The anterior drawer sign is performed to evaluate the anterior cruciate ligament; however the patient is supine, hips and knees flexed, and feet are flat on the table.
C
Lachman test
D
McMurray test
Hint:
The McMurray test is performed to evaluate medial and lateral meniscal tears while rotating the lower leg internally and externally.
Question 2 Explanation: 
The Lachman test is performed to evaluate the anterior cruciate ligament. The knee is placed in 15 degrees of flexion and external rotation of the hip.
Question 3
A 12 year-old female presents for a routine sports physical. The physical exam reveals asymmetry of the posterior chest wall on forward bending. This is the most striking and consistent abnormality of which of the following?
A
Spondylolysis
Hint:
Spondylolysis presents with limitation of lumbar flexibility and tight hamstring muscles.
B
Spondolisthesis
Hint:
Spondylolisthesis presents with reduced lumbar lordosis and sacral kyphosis.
C
Scoliosis
D
Herniated disc
Hint:
Herniated disc presents with lumbar muscle spasm and a positive straight leg test.
Question 3 Explanation: 
Asymmetry of the posterior chest wall on forward bending is the most striking and consistent abnormality in patients with idiopathic scoliosis.
Question 4
Physical exam findings in a 4 year-old child that include blue sclerae and recurrent fractures indicates which of the following?
A
Ehlers-Danlos syndrome
Hint:
Physical exam findings in Ehlers-Danlos include laxity and hypermobility of joints, mitral valve prolapse, and associated degenerative arthritis.
B
Marfan syndrome
Hint:
Children with Marfan syndrome have hypotonia, arachnodactyly, joint laxity and dislocations.
C
Achondroplasia
Hint:
Children with achondroplasia are below normal standards on growth charts. They have difficulty balancing their large heads when beginning to walk.
D
Osteogenesis imperfecta
Question 4 Explanation: 
Mild osteogenesis imperfecta presents with blue sclerae, history of recurrent fractures and presenile deafness.
Question 5
A 65 year-old female presents to the office with a six-month history of back pain. The patient states that she is shrinking and thinks she is about an inch shorter than she was a year ago. Serum parathyroid hormone, calcium, phosphorus, and alkaline phosphatase are all normal. Which of the following would you most likely see on the x-ray of her spine?
A
Radiolucent lesions
Hint:
Paget's disease of bone presents with bone pain, kyphosis, bowed tibias, large head, and deafness. The initial lesions are destructive and radiolucent. Paget's disease has a normal serum calcium and phosphate, but the serum alkaline phosphatase is elevated.
B
Demineralization
C
Chondrocalcinosis
Hint:
Chondrocalcinosis is the presence of calcium-containing salts in articular cartilage and is commonly seen in hyperparathyroidism, diabetes, hypothyroidism, and gout.
D
Subperiosteal resorption
Hint:
Hyperparathyroidism is frequently asymptomatic. Serum parathyroid hormone and serum calcium are elevated. X-ray findings include demineralization, subperiosteal resorption of bone especially in the radial aspects of the fingers.
Question 5 Explanation: 
Osteoporosis presents with varying degrees of back pain and loss of height is common. The serum calcium, parathyroid hormone, phosphorus, and alkaline phosphatase are normal. X- ray findings demonstrate demineralization in the spine and pelvis.
Question 6
In a trauma patient who has a suspected cervical spine injury, the x-ray view that will identify the majority of significant injuries is
A
lateral.
B
oblique
Hint:
The oblique view is usually not included in the initial set of x-rays taken. Bilateral supine oblique is a view that may be ordered if all seven cervical vertebrae are not seen on the lateral view.
C
anteroposterior
Hint:
Anteroposterior view shows < 1% of significant injuries.
D
odontoid.
Hint:
The odontoid view reveals 10% of significant injuries.
Question 6 Explanation: 
The lateral view shows 70-80% of significant injuries. It is important to visualize all seven cervical vertebrae and the upper margin of T1 to avoid missing possible pathology.
Question 7
A 38 year-old male sustained a fracture of the left distal tibia following a 25-foot fall and is taken to the operating room for an open reduction internal fixation of the distal tibia. Sixteen hours post-op, the patient develops sustained pain, which is not relieved with narcotics. On passive range of motion of the toes the patient "yells" in agony. The patient also states that the top of his foot has decreased sensation. On physical examination the physician assistant notes that the leg is swollen and the foot is cool to touch. Based upon this information what diagnostic testing should be done?
A
X-ray of the lower leg and ankle.
Hint:
X-rays of the lower leg and ankle will only determine bone placement.
B
Doppler studies.
Hint:
Doppler studies will confirm the presence of a decreased pulse.
C
Bone scan.
Hint:
A bone scan is not indicated in the evaluation of compartment syndrome.
D
Compartment pressure
Question 7 Explanation: 
Compartmental pressures should be obtained as soon as possible. If they are elevated this is a surgical emergency.
Question 8
A 32-year-old male presents with an acute onset of pain and swelling in his left ankle. On physical exam, the ankle is warm, swollen and erythematous. Evaluation of the synovial fluid reveals only leukocytosis with a low glucose. Which of the following is the most likely diagnosis?
A
Gout
Hint:
Gout and pseudogout are excluded by the failure to find crystals on synovial fluid analysis.
B
Pseudogout
Hint:
Gout and pseudogout are excluded by the failure to find crystals on synovial fluid analysis.
C
Acute rheumatic fever
Hint:
Acute rheumatic fever commonly involves multiple joints.
D
Septic arthritis
Question 8 Explanation: 
Leukocytosis and a low synovial glucose are indicative of septic arthritis.
Question 9
A 32-year-old male presents with migratory arthralgias and profound malaise and fatigue. He states that one week ago he returned from a hunting trip in Pennsylvania. He is also complaining of a lesion on his left thigh that he noticed about 3 days ago. Physical exam reveals a large annular lesion with a bright red outer border and partial central clearing. Which of the following is the most likely diagnosis?
A
Rheumatoid arthritis
Hint:
Rheumatoid arthritis is a symmetrical arthritis that commonly affects the proximal interphalangeal and metacarpophalangeal joints. It is not associated with the rash of erythema migrans.
B
Kawasaki disease
Hint:
Kawasaki disease is an acute febrile, multisystem disease of children. It is characterized by unresponsiveness to antibiotics, nonsuppurative cervical adenitis, and changes in the skin and mucous membranes such as edema, erythema of the lips and palms, and desquamation of the skin of the fingertips.
C
Lyme disease
D
Nongonococcal arthritis
Hint:
Nongonococcal arthritis occurs in patients with an underlying predisposition such as rheumatoid arthritis. The common presentation is involvement of a single joint.
Question 9 Explanation: 
After an incubation period of 3 to 32 days, erythema migrans develops at the site of the tick bite. Within days or weeks after the onset of erythema migrans the patients develop a severe headache, mild stiffness of the neck, migratory musculoskeletal pain, arthralgias and profound malaise and fatigue.
Question 10
A 22 year-old male presents with pain along the medial tibia. The pain initially began towards the end of soccer practice but now it is present earlier on. Physical exam reveals pain to palpation over the posterior tibialis muscle body. What is the most likely diagnosis?
A
Shin splint
B
Stress fracture
Hint:
See answer for explanation.
C
Osgood-Schlatter disease
Hint:
Osgood-Schlatter disease is an injury occurring at the insertion of the patellar tendon on the tibial tuberosity in a younger age group
D
Patellofemoral pain syndrome
Hint:
Patellofemoral pain syndrome is the most common cause of chronic anterior knee pain, more commonly seen in females.
Question 10 Explanation: 
Shin splints cause pain over the posterior tibialis muscle body as opposed to discrete pain over the tibia with a stress fracture.
Question 11
A patient who demonstrates pain on the radial aspect of the wrist with abrupt ulnar movements while the thumb is flexed into the closed palm most likely has
A
carpal tunnel syndrome.
Hint:
Carpal tunnel is diagnosed by a positive Tinel's or Phalen's maneuver.
B
radial tunnel syndrome.
Hint:
Radial syndrome is demonstrated by simultaneously extending the patient's wrists and fingers while the examiner passively flexes the patient's long finger, which causes pain.
C
tenosynovitis.
D
gamekeeper's thumb.
Hint:
Gamekeeper's thumb is the most common injury to the metacarpophalangeal collateral ligaments, causing a sprain.
Question 11 Explanation: 
Tenosynovitis is diagnosed using Finkelstein maneuver. The patient's thumb is placed in the palm of the hand and the wrist is abruptly deviated to the ulnar aspect of the wrist, causing pain on the radial aspect.
Question 12
The most important preventive medicine recommendation for patients with osteoarthritis is which of the following?
A
Start an exercise program
B
Brace the affected joint
Hint:
Bracing or resting of the affected joint in a patient with osteoarthritis may limit joint motion and restrict mobility, which may negatively impact the disease.
C
Rest the joint
Hint:
Bracing or resting of the affected joint in a patient with osteoarthritis may limit joint motion and restrict mobility, which may negatively impact the disease.
D
Inject steroids monthly
Hint:
Steroid injection into the affected joint may decrease pain but injections are limited to not more than three per year.
Question 12 Explanation: 
Patients with osteoarthritis who exercise are able to maintain range of motion, strengthen periarticular muscles, and improve physical fitness.
Question 13
Bone mass measurement should be considered in all women by what age?
A
30 - 35
Hint:
See answer for explanation.
B
40 - 45
Hint:
See answer for explanation.
C
50 - 55
Hint:
See answer for explanation.
D
60 - 65
Question 13 Explanation: 
According to the National Osteoporosis Foundation, all women should have a bone mass measurement by age 60 - 65.
Question 14
A 32 year-old medical transcriptionist presents with burning and tingling in her right wrist and hand for the past month. On physical exam, Phalen's test is positive; however, there is no atrophy of the thenar eminence. Which of the following is the initial step in management of this patient?
A
Wrist splint for 2-6 weeks
B
Corticosteroid injection
Hint:
Corticosteroid injections and surgery are indicated only after a trial of the wrist splint provides no relief.
C
Surgical referral
Hint:
Corticosteroid injections and surgery are indicated only after a trial of the wrist splint provides no relief.
D
Darvocet
Hint:
Darvocet has no role in the treatment of carpal tunnel syndrome
Question 14 Explanation: 
The treatment of carpal tunnel syndrome is aimed at relieving the pressure on the median nerve. This is best accomplished by having the patient wear a wrist splint during the activities that increase the pressure on the median nerve.
Question 15
Which of the following is the correct treatment for a Grade II ankle sprain resulting from an inversion injury?
A
Corticosteroid injection
Hint:
See answer for explanation.
B
Rest, ice, compression, elevation
C
Moist heat and a walking cast
Hint:
See answer for explanation.
D
Surgical intervention
Hint:
See answer for explanation.
Question 15 Explanation: 
The majority of ankle sprains are treated with RICE (rest, ice, compression and elevation). Corticosteroid injections, moist heat, and a walking cast provide no benefit. Surgical intervention for repair of a ruptured ligament is only necessary in chronically unstable joints.
Question 16
A 4 year-old boy presents to the ED after sustaining a crush injury to his distal third phalanx. Physical exam reveals an associated nail bed injury. Which of the following is the appropriate management?
A
Rest, ice, elevation
Hint:
Rest, ice, and elevation are only palliative measures and not appropriate management
B
Immobilize, antibiotics, orthopedics referral
C
Splint for 48 hours, aspirin, ice
Hint:
The digit must be immobilized until seen by ortho. Aspirin is not an appropriate analgesic for children.
D
Surgical referral for amputation of digit
Hint:
See answer for explanation.
Question 16 Explanation: 
Distal phalanx fracture should be immobilized and if there is an associated nailbed injury the fracture is considered "open" and the patient should be given antibiotics and follow-up with ortho in one week.
Question 17

Which of the following clinical characteristics is associated with bicipital tendonitis?

A
Aggravated by resisted supination of the forearm
B
Bulging appearance to the proximal arm
Hint:
Biceps rupture may present with a bulging appearance of the proximal arm
C
Weakness of the arm with internal rotation and adduction
Hint:
Weakness of the arm with internal rotation and adduction is characteristic of pectoralis major rupture or tear.
D
Pain that awakens the patient at night
Hint:
Night pain is characteristic of rotator cuff tear or tendonitis.
Question 17 Explanation: 
Bicipital tendonitis will be aggravated by resisted supination of the forearm.
Question 18

Which of the following risk factors is the most predictive for the development of osteoarthritis?

A
Age
B
Major joint trauma
Hint:
See answer for explanation.
C
Prior inflammatory joint disease
Hint:
See answer for explanation.
D
Repetitive stress
Hint:
See answer for explanation.
Question 18 Explanation: 
Age is the most significant risk factor for osteoarthritis. Prevalence and severity increase with age. Ninety percent of people greater than the age of 40 have degenerative changes of the weight bearing joints. Major joint trauma, prior inflammatory joint disease, and repetitive stress are additional risk factors but not as predictive as age.
Question 19
The most accurate way to determine the exact degree of spinal curvature in a child with scoliosis is by which of the following?
A
Calculation of the Cobb angle
B
Measurement of waist asymmetry
Hint:
While waist asymmetry and rib hump deformity may be observed on physical examination of the patient with scoliosis, none of them can be used to determine the exact degree of the spinal curvature.
C
Measurement of rib hump deformity
Hint:
While waist asymmetry and rib hump deformity may be observed on physical examination of the patient with scoliosis, none of them can be used to determine the exact degree of the spinal curvature.
D
Calculation using a scoliometer
Hint:
A scoliometer or inclinometer measures distortions of the torso and is good for screening angle of rotation, but is not exact to determine exact degree of curvature.
Question 19 Explanation: 
The scoliotic curve is measured by the Cobb method using AP and lateral x-ray films of the entire length of the spine.
Question 20
A 53 year-old female massage therapist presents with new onset of sudden swelling involving the right elbow. She denies previous episodes similar to this. On physical examination, the patient is afebrile. There is a 4 cm fluid-filled mass that is tender to palpation overlying the tip of the elbow with no evidence of erythema or warmth. Which of the following is the most appropriate intervention?
A
Rest and NSAIDs
B
Surgical excision
Hint:
Surgical excision is reserved for chronic bursitis and is not indicated in this acute initial presentation.
C
Incision and drainage
Hint:
Incision and drainage is not recommended as it may cause a chronic drainage sinus tract.
D
Aspiration and corticosteroid injection
Hint:
Aspiration of the bursa and corticosteroid injection are second-line therapy in a patient with olecranon bursitis who fails rest and NSAIDs.
Question 20 Explanation: 
Rest and NSAID is the most appropriate initial intervention in a patient with olecranon bursitis.
Question 21
A 46-year-old male presents to the ER with a limp and pain in the back of his right lower leg. He recounts an unusual incident during his game earlier today. While attempting to make a rapid move to outpace an opponent, he experienced a sudden, sharp pain in the back of his right lower leg. He describes it as feeling as if he'd been kicked, but he is sure there were no players behind him at that moment. There is no significant swelling, but he complains of difficulty putting weight on the affected leg. What is the most likely diagnosis?
A
Quadriceps tendon rupture
Hint:
This injury typically presents with sudden pain in the anterior aspect of the knee or thigh, especially during a forceful contraction of the muscle (like jumping). It often prevents the patient from extending the knee, which doesn't match this patient's symptoms.
B
Hamstring strain
Hint:
While a hamstring strain can occur during activities like soccer, it generally causes pain at the back of the thigh, not the lower leg. Hamstring injuries are also associated with a "popping" sensation at the time of injury, which was not reported by this patient.
C
Achilles tendon rupture
D
Gastrocnemius muscle tear
Hint:
Also known as "tennis leg", it is caused by a sudden forceful contraction of the calf muscles, such as during a sprint or jump. While it can cause acute pain in the calf, patients typically don't describe the "kicked in the leg" sensation that is characteristic of an Achilles tendon rupture. Instead, they often report a "pop" in the calf at the time of injury.
Question 21 Explanation: 
The patient's symptoms of sudden, sharp pain in the back of the lower leg during a fast, forceful movement, and difficulty bearing weight on the affected leg are typical of an Achilles tendon rupture. This condition is often described as feeling like a "kick in the back of the leg" with no one behind you.
Question 22
During the stages of fracture healing which of the following is responsible for producing collagen?
A
Osteoclasts
Hint:
Osteoclasts are responsible for removing necrotic bone.
B
Chondrocytes
Hint:
Chondrocytes make up the articular cartilage.
C
Glycosaminoglycans
Hint:
Glycosaminoglycans help form the osteon or vascular canal.
D
Fibroblasts
Question 22 Explanation: 
Fibroblasts produce collagen during the inflammation stage of healing.
Question 23

Which of the following medications inhibits prostaglandin synthesis in a patient with rheumatoid arthritis?

A
Methotrexate
Hint:
Methotrexate inhibits the enzyme dihydrofolate reductase.
B
Infliximab (Remicade)
Hint:
Infliximab (Remicade) neutralizes cytokine tumor necrosing factor.
C
Probenecid (Benemid)
Hint:
Probenecid blocks the tubular reabsorption of filtered urate and is used to reduce serum uric acid.
D
Aspirin
Question 23 Explanation: 
Salicylates inhibit the enzymatic production of prostaglandins by inhibiting cyclooxygenase.
Question 24

Which of the following mechanisms of action is most commonly associated with meniscal tears?

A
Hyperextension
Hint:
Hyperextension injuries usually result in ACL and PCL injuries.
B
Axial loading and rotation
C
Hyperflexion
Hint:
Hyperflexion injuries result in PCL injuries.
D
Valgus force to the lateral knee
Hint:
Valgus force to the lateral knee more than likely results in medial collateral ligament injuries.
Question 24 Explanation: 
Axial loading and rotation most likely result in meniscal injuries.
Question 25
A 14 year-old patient, who fell on his outstretched hand, complains of pain along his entire arm. There is point tenderness and swelling over the midshaft of the radius. There is significant pain with limited flexion of the elbow joint. An x-ray will most likely show which of the following fractures?
A
Galeazzi's
B
Scaphoid
Hint:
A scaphoid fracture is a fracture of the scaphoid bone and would not cause pain in the elbow joint.
C
Colles'
Hint:
A Colles' fracture of the distal radius has a characteristic "silver fork" deformity, but does not involve the elbow joint.
D
Smith's
Hint:
A Smith's fracture is the reverse of a Colles' fracture, with volar angulation of the distal radius, but does not involve the elbow joint.
Question 25 Explanation: 
Galeazzi's fracture/dislocation involves a fracture of the mid or distal radial shaft with distal radioulnar joint dislocation.
Question 26
Which of the following medications used to treat rheumatoid arthritis is contraindicated in patients with chronic hepatitis?
A
Sulfasalazine
Hint:
Sulfasalazine is a second line medication that can cause neutropenia and thrombocytopenia.
B
Methotrexate
C
Minocycline
Hint:
Minocycline is used for early rheumatoid arthritis with minimal adverse effects.
D
Infliximab
Hint:
Infliximab is a tumor necrosing factor inhibitor and should be used cautiously in patients with heart failure.
Question 26 Explanation: 
Methotrexate is contraindicated in patients with chronic hepatitis.
Question 27

Which of the following x-ray views will show the presence of a "Scotty dog" deformity seen with spondylolysis?

A
Lateral
Hint:
The lateral view is the most appropriate for evaluation of the possible presence of spondylolisthesis, not spondylolysis.
B
Oblique
C
Anteroposterior
Hint:
Anteroposterior views will show the alignment of the spinous processes, but not a defect in the pars interarticularis.
D
Open-mouth odontoid
Hint:
The open-mouth odontoid view is used to visualize the odontoid process and the relationship between the C1 and C2 vertebrae.
Question 27 Explanation: 
Spondylolysis results from a defect through the pars interarticularis, which is seen as a defect in the neck of the "Scotty dog" on the oblique view.
Question 28
An x-ray taken on a patient complaining of wrist pain after being hit by a baseball reveals a non-displaced mid-shaft ulnar fracture. Which of the following splints is most appropriate for treatment?

 

A
Thumb spica
Hint:
Thumb spica splints are used for scaphoid fractures.
B
Sugar tong
C
Cock-up wrist
Hint:
Cock-up wrist splints may be useful in some situations unrelated to fractures, such as to immobilize the wrist for tendinitis or to support it in the case of wrist drop due to radial nerve palsy but not to be used in wrist fractures.
D
Short arm gutter
Hint:
Short arm gutter splints immobilize only the wrist and the ulnar or radial half of the hand.
Question 28 Explanation: 
Sugar tong splints are best used to immobilize the elbow, wrist and forearm.
Question 29
A 12-year-old male presents with pain in his left leg that is worse at night. Aspirin relieves the pain and the patient denies injury. On examination, there is point tenderness over the tibia, and the patient has a slight limp that favors the left leg. Radiographs show a 1 cm radiolucent nidus surrounded by osteosclerosis seen here. Which of the following is the most likely diagnosis?
A
Osteosarcoma
Hint:
Osteosarcoma and Ewing sarcoma are malignant bone tumors that present with pain and swelling. No improvement is noted with conservative therapy.
B
Legg-Calve-Perthes disease
Hint:
Legg-Calve-Perthes disease is avascular necrosis of the hip affecting boys ages 4-10.
C
Osgood-Schlatter disease
Hint:
Osgood-Schlatter disease is inflammation of the tibial tuberosity affecting mainly boys in the ages of 10-15. Commonly associated bilaterally and due to jumping.
D
Osteoid osteoma
Question 29 Explanation: 
Osteoid osteoma is a benign tumor in children age 5 to 20, presents with increasing pain, worse at night and relieved by aspirin.
Question 30

Which of the following interventions is initially indicated for helping to relieve the symptoms of plantar fasciitis?

A
Steroid injections
Hint:
Steroid injections can be used to relieve symptoms but may be harmful by causing rupture if given in the plantar tendon greater than 3 or 4 times.
B
Short leg walking cast
Hint:
Short leg walking cast may be used for severe cases refractory to initial treatment.
C
Arch supports
D
Surgical release
Hint:
Surgical release of the plantar tendon is usually reserved for patients who fail all therapies.
Question 30 Explanation: 
Arch supports, NSAIDs and stretching exercises are the initial interventions to help relieve symptoms.
Question 31
When injecting a corticosteroid into a joint to help relieve pain, which of the following would most commonly be mixed in the syringe?
A
Hyaluronic acid
Hint:
Hyaluronic acid is usually injected by itself in patients with osteoarthritis.
B
Ketorolac
Hint:
Ketorolac is an NSAID that is given orally and not intra-articularly.
C
Chondroitin sulfate
Hint:
Chondroitin sulfate is taken orally and thought to improve symptoms in osteoarthritis.
D
Lidocaine
Question 31 Explanation: 
1% lidocaine and corticosteroids are most commonly used together for joint injections.
Question 32
A 35 year-old female presents with a long standing complaint of dry, scratchy eyes and dry mouth. She also reports dyspareunia. Labs demonstrate a positive anti-nuclear antibody and Anti-La antibodies. The patient has a prolonged Schirmer test. Which of the following classes of medicines should be avoided in treating this condition?
A
Antimalarials
Hint:
See answer for explanation.
B
NSAIDs
Hint:
See answer for explanation.
C
Glucocorticoids
Hint:
See answer for explanation.
D
Anticholinergics
Question 32 Explanation: 
Anticholinergics should be avoided in a patient with Sjogrens syndrome. All the other medications may be used to treat the systemic symptoms of Sjogrens.
Question 33

In the neonate, unequal thigh folds may indicate which of the following?

A
Coxa vara
Hint:
Coxa vara is a hip deformity that would present with a decrease in the hip angle and a shift of the femoral shaft medially.
B
Legg-Calve-Perthes disease
Hint:
Legg-Calve-Perthes disease presents with a painless limp in children ages 4-10 due to avascular necrosis of the femoral head.
C
Developmental hip dysplasia
D
Slipped capital femoral epiphysis
Hint:
A slipped capital femoral epiphysis is primarily an adolescent disorder with decreased range of motion in abduction and internal rotation of the hip on physical examination.
Question 33 Explanation: 
A dislocated hip displaces proximally in developmental hip dysplasia, causing a shortening of the leg that may present as unequal thigh folds.
Question 34
Which of the following is the underlying pathogen for the development of Lyme disease?
A
Borrelia burgdorferi
B
Bartonella henselae
Hint:
Bartonella henselae is the spirochete that causes cat scratch fever.
C
Rickettsia rickettsii
Hint:
Rickettsia Rickettsii is the spirochete that causes Rocky Mountain Spotted Fever.
D
Coxiella brunetti
Hint:
Coxiella brunetti is the spirochete that causes Q fever.
Question 34 Explanation: 
Lyme disease is a tick-borne illness caused by the spirochete B. burgdorferi.
Question 35
Which of the following is an extra-articular manifestation of rheumatoid arthritis?
A
Vasculitis
B
Malar rash
Hint:
A malar rash is frequently seen in systemic or discoid lupus.
C
Coronary artery aneurysms
Hint:
Coronary artery aneurysms are frequently a manifestation of Kawasaki's disease.
D
Periorbital xanthelasma
Hint:
Xanthelasma is a skin finding that occurs in hyperlipidemia.
Question 35 Explanation: 
Vasculitis affecting any organ system is seen in patients with severe rheumatoid arthritis.
Question 36
Which of the following groups is most likely to present with Duchenne's muscular dystrophy?
A
Adolescent females
Hint:
See answer for explanation.
B
Middle-aged males
Hint:
See answer for explanation.
C
Infant females
Hint:
See answer for explanation.
D
Toddler-aged males
Question 36 Explanation: 
Duchenne's muscular dystrophy, a genetic defect on the short arm of the X chromosome, affects toddler-aged males.
Question 37

Spina bifida occulta is usually detected by which initial diagnostic evaluation?

A
Electroencephalogram
Hint:
Electroencephalogram (EEG) cannot detect or diagnose spina bifida occulta.
B
Alpha-fetoprotein levels
Hint:
Alpha-fetoprotein, measured at 16-18 weeks of pregnancy, if elevated, would indicate a neural tube defect, such as spina bifida.
C
Folic acid levels
Hint:
Folic acid has been shown to decrease the incidence of neural tube defects, not as an aid in diagnosis of the defect.
D
X-ray of the spine
Question 37 Explanation: 
X-ray or MRI is the definitive test to diagnose spina bifida occulta, showing the vertebral bony defect.
Question 38
A mother brings her 14 month-old son to your clinic. Earlier today she lifted her son by grabbing him by the wrists and pulling him up off the floor. The child is sitting in his mother's lap with his left forearm is extended and in pronation. He is refusing to move the left arm, forearm or wrist. The arm and joints appear normal with no noted deformities, edema or erythema. Distal pulses and capillary refill are normal and he can move his fingers. Which of the following is the most likely diagnosis?
A
Nursemaid's elbow
B
Fractured left wrist
Hint:
There is no edema, deformity or erythema to suggest a fractured wrist.
C
Osteochondritis dissecans
Hint:
Osteochondritis dissecans is avascular necrosis of subchondral bone, most commonly seen during adolescence.
D
Child abuse
Hint:
Although child abuse could be suspected the clinical history best describes nursemaid's elbow.
Question 38 Explanation: 
This clinical history is classic for radial head dislocation or nursemaid's elbow.
Question 39
A patient presents with chronic back pain. On physical examination testing, the patient is found to have abnormalities of proprioception and vibration discrimination. Which of the following portions of the spinal column are most likely affected?  
A
Lateral spinothalamic tract
Hint:
The lateral spinothalamic tract affects pain and temperature sensation.
B
Ventral spinothalamic tract
Hint:
The ventral spinothalamic tract affects pressures and touch sensations.
C
Posterior column
D
Transection of the cord
Hint:
Patients with transection of the cord will have loss of sensation distal to the area of injury along with paralysis and hyperactive reflexes in the area distal to the transection.
Question 39 Explanation: 
The posterior column affects proprioception (position sense) and vibration sense.
Question 40
In adults and intravenous drug abusers, which of the following bones is most commonly affected with acute osteomyelitis?
A
Femur
Hint:
Long bones are most commonly affected with osteomyelitis in children.
B
Humerus
Hint:
Long bones are most commonly affected with osteomyelitis in children.
C
Vertebral spine
D
Tibia
Hint:
Long bones are most commonly affected with osteomyelitis in children.
Question 40 Explanation: 
The bones of the vertebral spine are most commonly affected in a patient with osteomyelitis. Organisms reach the well-perfused vertebral body of adults via spinal arteries and quickly spread from the end plate into the disk space and then to the adjacent vertebral body. The infection may originate in the urinary tract and intravenous drug use carries an increased risk of spinal infection
Question 41
Which of the following medications is the treatment of choice for patients with chronic gout to prevent recurrence of symptoms during its quiescent phase?
A
Probenecid (Benemid)
Hint:
Probenecid is a uricosuric medication that helps to increase the excretion of uric acid but it does not prevent the formation of uric acid making it less beneficial in chronic gout therapy. It is also not effective in patients with chronic renal disease.
B
Allopurinol (Zyloprim)
C
Colchicine
Hint:
Colchicine treatment is recommended only in patients who have tophaceous deposits in the skin and is used in the acute rather than the chronic setting of gout.
D
Indomethacin (Indocin)
Hint:
Indomethacin is used in the acute management of gout but is not effective in decreasing monosodium urate deposition in the joints.
Question 41 Explanation: 
Allopurinol is the best drug to lower serum urate in overproducers, stone formers, and patients with advanced renal failure. It is a xanthine oxidase inhibitor that is used to prevent the formation of uric acid.
Question 42
Intraarticular injection of hyaluronic acid has been approved for treatment of patients with which of the following conditions?
A
Rheumatoid arthritis of the knee
Hint:
Intraarticular injection of hyaluronic acid has been approved recently for treatment of patients with osteoarthritis of the knee that have failed other therapies. Although the onset of action of this medication is slower than injected glucocorticoids, it has a sustained length of activity outlasting the injected glucocorticoids.
B
Osteoarthritis of the knee
C
Olecranon bursitis
Hint:
Treatment of olecranon bursitis may include incision and drainage but not hyaluronic acid injections.
D
Gouty arthritis
Hint:
Gout is treated with anti-inflammatory medications.
Question 42 Explanation: 
Intraarticular injection of hyaluronic acid has been approved recently for treatment of patients with osteoarthritis of the knee that have failed other therapies. Although the onset of action of this medication is slower than injected glucocorticoids, it has a sustained length of activity outlasting the injected glucocorticoids.
Question 43
A 25 year-old presents with pain in the proximal ulna after falling directly on the forearm. X-ray shows fracture of the proximal 1/3rd of the ulna. There is an associated anterior radial head dislocation. What is the proper name for this condition?
A
Galeazzi fracture
Hint:
A Galeazzi fracture is a fracture along the length of the radius with injury to the distal radioulnar joint.
B
Monteggia fracture
C
Colles' fracture
Hint:
A Colles' fracture is a fracture of the distal radius with dorsal displacement of the radial head.
D
Smith fracture
Hint:
A Smith fracture is a fracture of the distal radius with ventral displacement of the radial head.
Question 43 Explanation: 
A Monteggia fracture is a fracture of the proximal ulna with anterior dislocation of the radial head.
Question 44
A 75 year-old female presents with medial knee pain that worsens with stair climbing. Physical examination reveals swelling and point tenderness inferior and medial to the patella and tenderness overlying the medial tibial plateau. Which of the following is the most likely diagnosis?
A
Pes anserine bursitis
B
Prepatellar bursitis
Hint:
Prepatellar bursitis causes swelling in the prepatellar area and is worse with kneeling. The prepatellar bursa is superficial and is located over the inferior portion of the patella.
C
Infrapatellar bursitis
Hint:
The infrapatellar bursa is deeper and lies beneath the patellar ligament before its insertion on the tibial tubercle. It has a midline location rather than the medial surface as described in the question.
D
Trochanteric bursitis
Hint:
Trochanteric bursitis causes hip pain rather than knee pain.
Question 44 Explanation: 
The pes anserine bursa underlies the semimembranosus tendon and may become inflamed or painful owing to trauma, overuse, or inflammation. It is a common cause of knee pain and it is often misdiagnosed in adults.
Question 45
A 28-year-old female, who has experienced occasional painful migratory arthralgias, complains now of a tender, swollen, and hot left ankle. The joint was aspirated and the synovial fluid showed 55,000 WBCs, 75% polymorphonuclear leukocytes, low glucose level, and no crystals. Which of the following would be the most likely diagnosis?
A
Rheumatoid arthritis
Hint:
Rheumatoid arthritis usually involves more than one joint, primarily affecting metacarpophalangeal and wrist joints initially. Synovial fluid analysis would show moderate leukocytosis (< 50,000) with higher glucose levels.
B
Septic arthritis
C
Gouty arthritis
Hint:
Although gout presents as monarticular process, it usually presents acutely without previous migratory arthralgias. Crystals are usually found in synovial analysis.
D
Osteoarthritis
Hint:
Osteoarthritis effusions may be normal or show mild elevations of WBCs in the fluid analysis, but it is rarely monarticular.
Question 45 Explanation: 
Septic arthritis presents with a large number of WBCs, predominantly polymorphonuclear, and with glucose levels much lower than serum levels.
Question 46
A 44 year-old female presents with ongoing arthralgias and myalgias with intermittent flares of arthritis. She is found to have a malar rash that worsens with sun exposure. She is known to have progressive renal damage and has recurrent infections that are slow to respond to therapy. She takes ibuprofen (Motrin) as needed for her joint pain and takes no other medication. Which of the following tests would be the initial test recommended to screen for this diagnosis?
A
Rheumatoid factor
Hint:
Rheumatoid factor is most commonly performed in the assessment of rheumatoid arthritis and not suspected systemic lupus erythematosus.
B
Antihistone antibodies
Hint:
Antihistone antibodies are seen in drug-induced SLE rather than in spontaneously occurring SLE.
C
Anti-Smith (Anti-Sm) antibodies
Hint:
Anti-Smith antibodies and antibodies to the double stranded DNA are fairly specific to SLE but their role is for confirmation rather than screening for SLE.
D
Anti-nuclear antibodies (ANA)
Question 46 Explanation: 
ANA's are the best screening test used in the evaluation for SLE.
Question 47
A 53 year-old patient presents with severe pain at the base of the thumb and no other finger involvement. The pain is worse with activity and lasts a short period of time following rest. There is no specific history of trauma to the thumb but the patient admits working with her hands as a typist. Which of the following is the most likely diagnosis?
A
Rheumatoid arthritis
Hint:
Rheumatoid arthritis typically involves the MCP and PIP joints of the digits for the second through fifth fingers. The thumb is classically spared.
B
Osteoarthritis
C
Hemochromatosis
Hint:
Hemochromatosis classically involves the MCP joints of the second through fifth fingers.
D
Pseudogout
Hint:
Pseudogout joint involvement is typically the MCP joints of the second through fifth fingers.
Question 47 Explanation: 
The base of the thumb is typically involved with osteoarthritis as are the DIP joints of the other fingers.
Question 48
Which of the following is the treatment of choice for a torus (buckle) fracture involving the distal radius?
A
Open reduction and internal fixation
Hint:
See answer for explanation.
B
Ace wrap or anterior splinting
C
Closed reduction and casting
Hint:
See answer for explanation.
D
Corticosteroid injection followed by splinting
Hint:
See answer for explanation.
Question 48 Explanation: 
A torus or buckle fracture occurs after a minor fall on the hand. These fractures are very stable and are not as painful as unstable fractures. They heal uneventfully in 3-4 weeks.
Question 49
An 80 year-old female presents with pain in her vertebral column. Radiography reveals compression fracture of T12 that is consistent with osteoporotic compression fracture. Which of the following treatment modalities has the potential to cause analgesia of the fracture site with its use?
A
Calcitonin (Miacalcin) nasal spray
B
Alendronate (Fosamax)
Hint:
Alendronate is effective in building new bone for a patient with osteoporosis but has no associated analgesic effect.
C
Raloxifene (Evista)
Hint:
Raloxifene is a selective estrogen receptor modulator and has positive effects on bone density when used to treat osteoporosis. Raloxifene, however, has no analgesic properties.
D
Combined estrogen and progesterone (Prempro) therapy
Hint:
Combined hormonal therapy may have positive effects on bone density but it has no analgesic properties.
Question 49 Explanation: 
Calcitonin has the ability to cause analgesia when used for acute compression fracture of the vertebral body
Question 50
Which of the following views on plain films is preferred to identify spondylolysis?
A
Anterior
Hint:
See answer for explanation.
B
Posterior
Hint:
See answer for explanation.
C
Oblique
D
Lateral
Hint:
See answer for explanation.
Question 50 Explanation: 
The defect in the pars articularis (usually bilateral) is best visualized on the oblique projections on plain films.
Question 51
A 15 year-old softball player presents after jamming the distal tip of her finger into severe flexion. She is unable to extend the distal phalanx and she has pain on palpation of the distal interphalangeal joint. X-ray of the hand fails to reveal any associated avulsion fracture. Which of the following is the treatment of choice?
A
Open reduction and internal fixation
Hint:
The treatment of choice for a tear in the extensor tendon of the finger is continuous extension of the DIP via splinting for 6 to 8 weeks
B
Continuous extension of the DIP with splinting
C
Continuous flexion of the PIP with splinting
Hint:
See answer for explanation.
D
Application of short arm cast
Hint:
Short arm casting is indicated in wrist and metacarpal injuries but not in DIP extensor injuries.splinting for 6 to 8 weeks
Question 51 Explanation: 
The treatment of choice for a tear in the extensor tendon of the finger is continuous extension of the DIP via splinting for 6 to 8 weeks
Question 52
A 75 year-old female falls on her outstretched arm. She sustains a humeral mid-shaft fracture. Nerve impingement occurs due to the fracture. What is the most likely physical examination abnormality that will be encountered?
A
Inability to extend the wrist against resistance
B
Numbness over the deltoid muscle in the shoulder
Hint:
Axillary nerve injury results in numbness over the deltoid muscle; this nerve is more commonly injured in proximal humeral fractures and anterior shoulder dislocations.
C
Winging of the scapula
Hint:
Injury to the long thoracic nerve causes winging of the scapula due to its innervation of the serratus anterior muscle.
D
Weakness of the rotator cuff
Hint:
Injury to the subscapular nerve results in weakness and pain of the infraspinatus muscle; this injury is commonly seen in volleyball players from repetitive stress.
Question 52 Explanation: 
The radial nerve is most likely entrapped by this fracture. Radial nerve damage will cause an inability to extend the wrist against resistance.
Question 53
A 55 year-old female presents with complaints of stiffness, aching, and pain in the muscles of her neck, shoulders, lower back, hips, and thighs. There is no associated weakness associated with the stiffness and achiness. Laboratory evaluation shows an elevated C reactive protein and erythrocyte sedimentation rate. Which of the following medications is used to treat this condition immediately and will also serve to prevent a known complication from this disorder?
A
Glucocorticoids
B
Cyclophosphamide (Cytoxan)
Hint:
Cyclophosphamide is an immunosuppressant used in the treatment of acute leukemia.
C
Methotrexate (Rheumatrex)
Hint:
Methotrexate is a folate inhibitor used to treat rheumatoid arthritis, not polymyalgia rheumatica.
D
Azathioprine (Imuran)
Hint:
Azathioprine is an immunosuppressant that is used to treat rheumtatic disease and inflammatory bowel disease, not polymyalgia rheumatica.
Question 53 Explanation: 
This patient has polymyalgia rheumatica and treatment with glucocorticoids can relieve discomfort and prevent the associated ischemic temporal arteritis, which threatens vision.
Question 54
An 18 year-old male presents with pain in his wrist after he fell off of a moving motor cycle. Physical examination reveals tenderness in the anatomic snuffbox. No fracture is noted on plain radiography of the wrist. Which of the following is the recommended treatment for this patient?
A
Ace wrap of the wrist
Hint:
See answer for explanation.
B
Closed reduction of the fracture site
Hint:
See answer for explanation.
C
Thumb spica cast application
D
Open reduction of the fracture site
Hint:
See answer for explanation.
Question 54 Explanation: 
Even with normal initial radiographs, patients with a consistent history and tenderness in the anatomical snuffbox are treated as a stable fracture with immobilization in a thumb spica cast. Casting is recommended for all presumed nondisplaced scaphoid fractures.
Question 55
A 53-year-old male is seen in the emergency department following a motor vehicle collision in which his knee impacted against the dashboard. The patient has a posterior tibiofemoral dislocation that is promptly reduced in the emergency department. The patient currently has a palpable pulse in the dorsalis pedis and posterior tibial areas. Which of the following studies is mandatory?
A
Anterior plain film of knee
Hint:
See answer for explanation.
B
Sunrise view of the knee
Hint:
See answer for explanation.
C
Measurement of compartment pressures
Hint:
Compartment pressures are performed in cases of suspected compartment syndrome, not to determine the patency of the popliteal artery.
D
Angiography
Question 55 Explanation: 
The popliteal artery is at risk for injury whenever a patient sustains a posterior dislocation of the knee and should be evaluated with an arteriogram despite the presence of pedal pulses.
Question 56
Which of the following preventive strategies against osteoporosis-associated vertebral fractures has a known side effect of increasing the incidence of hot flashes when used in a perimenopausal female?
A
Calcitonin (Miacalcin) nasal spray
Hint:
Calcitonin does not have any estrogen effects on the body and serves as an analgesic when used in the management of vertebral fractures.
B
Alendronate (Fosamax)
Hint:
Alendronate is a bisphosphonate that does not have any hormonal effects on the body.
C
Estrogen/progesterone (Prempro)replacement
Hint:
Estrogen/progesterone replacement has the benefit of maintaining bone and decreasing vertebral fractures but it would improve perimenopausal hot flashes.
D
Raloxifene (Evista)
Question 56 Explanation: 
Raloxifene has effects on bone turnover and bone mass and has been shown to decease vertebral fractures. It has anti-estrogen effects on the non-skeletal portions of the body and increases hot flashes in perimenopausal females.
Question 57
A 65 year-old male presents with back pain two days after he was shoveling snow. The patient complains of pain in his low back that radiates into his buttocks, posterior thigh and calf, and the bottom of his foot. There is associated numbness of his lateral and plantar surface of his foot. Which of the following disc herniations is most likely to be affected?
A
L3-L4
Hint:
See answer for explanation.
B
L4-L5
Hint:
See answer for explanation.
C
L5-S1
D
S1-S2
Hint:
See answer for explanation.
Question 57 Explanation: 
The S1 nerve root impingement is most likely to occur from the herniation of the L5-S1 disc space. The S1 disc affects Achilles' reflex, the gastrocnemius and soles muscles, and the abductor hallucis and gluteus maximus muscles.
Question 58
Endotracheal intubation should be performed with caution in patients with which of the following underlying conditions due to the propensity to cause subluxation of C1 on C2?
A
Rheumatoid arthritis
B
Osteoarthritis
Hint:
Although patients with osteoarthritis may have neck pain and stiffness, there is no predilection for the atlantoaxial joints.
C
Gout
Hint:
Patients with gout are likely to have involvement of peripheral joints rather than spinal joints.
D
Pseudogout
Hint:
Patients with pseudogout are more likely to have involvement of the knees, wrist, shoulder, ankle, elbow, and hands rather than the cervical spine.
Question 58 Explanation: 
Patients with advanced rheumatoid arthritis will have synovitis of the atlantoaxial joint (C1-C2) which may damage the transverse ligament of the atlas, producing forward displacement of the atlas on the axis (atlantoaxial subluxation).
Question 59
A 55 year-old secretary presents with ongoing pain and numbness in her hand. These symptoms are worse at night and she must shake her hand to regain feeling in it. Which of the following physical examination signs will be present?
A
Hypothenar atrophy
Hint:
Hypothenar atrophy may occur with aging and disuse but it is not part of the median nerve involvement that occurs with carpal tunnel syndrome.
B
Weakness of finger abduction
Hint:
Finger abduction weakness is associated with ulnar nerve injury, which does not occur with carpal tunnel syndrome.
C
Inability to maintain wrist extension against resistance
Hint:
Radial nerve injury causes weakness of wrist extension and this is not part of carpal tunnel syndrome.
D
Weakness of thumb abduction
Question 59 Explanation: 
Median nerve injury causes weakness of thumb abduction (measured by thumb opposition strength) along with thenar atrophy. Tinel's and Phalen's signs will also be positive with carpal tunnel syndrome.
Question 60
Which of the following rotator cuff tendons is most likely to sustain injury because of its repeated impingement (impingement syndrome) between the humeral head and the undersurface of the anterior third of the acromion and coracoacromial ligament?
A
Supraspinatus
B
Infraspinatus
Hint:
See answer for explanation.
C
Teres minor
Hint:
See answer for explanation.
D
Subscapularis
Hint:
See answer for explanation.
Question 60 Explanation: 
A critical zone exists for the supraspinatus tendon due to its superior insertion site. It is susceptible for injury because it has a reduction in its blood supply that occurs with abduction of the arm. Impingement of the shoulder is most commonly seen with the supraspinatus tendon, the long head of the biceps tendon and/or the subacromial bursa.
Question 61
Which of the following is the treatment of choice for stage one Lyme disease in a patient less than 12 years of age?
A
doxycycline
Hint:
See answer for explanation.
B
amoxicillin
C
chloramphenicol
Hint:
Chloramphenicol is only indicated in disseminated disease with known drug resistance. It is not indicated as first line therapy due to its tendency to cause aplastic anemia.
D
azithromycin
Hint:
Azithromycin is not as effective as doxycycline or amoxicillin.
Question 61 Explanation: 
Amoxicillin is first line therapy in a patient less than 12 years of age due to the harmful effects of doxycycline on teeth and bones in children.
Question 62
Which of the following treatment strategies is most likely to promote optimal health and limit the morbidity and mortality associated with rheumatoid arthritis?
A
meningococcal vaccination
Hint:
RA patients are at increased risk of infection and should receive influenza and pneumococcal vaccines, not meningococcal.
B
NSAIDs
Hint:
NSAIDs provide only symptomatic therapy and do not stop the disease progression of RA.
C
methotrexate
D
corticosteroids
Hint:
Chronic corticosteroid use is associated with greater toxicity than DMARDs and should be used only for symptomatic therapy.
Question 62 Explanation: 
Methotrexate is the most commonly prescribed DMARD for RA and has been proven to induce remission in some patients.
Question 63
Which of the following leads to retropatellar pain?
A
increased Q angle
B
increased quadriceps tone and strength
Hint:
Quadriceps strengthening aids in restoring patellofemoral tracking.
C
Osgood-Schlatter Disease
Hint:
Osgood-Schlatter disease involves the tibial tubercle, not the patellofemoral joint.
D
hamstring stretching
Hint:
Hamstring stretching aids in restoring patellofemoral tracking.
Question 63 Explanation: 
Patients with an increased Q angle have more force directed laterally during knee flexion and are at greater risk of patellofemoral pain syndrome.
Question 64
A 34 year-old female, with a past medical history of irritable bowel syndrome and migraines, presents with fatigue, generalized aching and stiffness of the trunk, hip, and shoulder girdles. She complains of pain and tightness in the neck and across the upper posterior shoulders. She complains of poor sleep, but denies depression. Physical examination is unremarkable except for numerous tender points on palpation. Laboratory evaluation was unremarkable. Which of the following is the best intervention for this patient?
A
corticosteroids
Hint:
Corticosteroids have no role in the management of fibromyalgia.
B
supervised exercise program
C
hydrocodone (Vicodin)
Hint:
Opioids are not first-line agents in the treatment of fibromyalgia.
D
refer to endocrinologist
Hint:
One-third of patients with fibromyalgia are found to have deficiency of growth hormone, but referral to endocrinology is not indicated at this time.
Question 64 Explanation: 
A carefully planned and individualized exercise program has been proven effective for the management of fibromyalgia.
Question 65
A 14 year-old male who is overweight presents with complaints of left knee and anteromedial thigh pain for the past month. He states the pain gets better with rest and denies any known trauma. On examination of the gait, a slight limp is noted. X-ray films of the left knee are normal. The most likely diagnosis is
A
genu valgum.
Hint:
Genu valgum is a knock-knee deformity of the knees and would be detected on physical examination.
B
Legg-Calve-Perthes disease.
Hint:
Legg-Calve-Perthes presents in a younger population.
C
Osgood-Schlatter disease.
Hint:
Osgood-Schlatter disease is characterized by local pain, swelling, and tenderness to palpation overlying the tibial tubercle, and x-ray findings of tibial tubercle prominence, with or without free bony fragments.
D
slipped capital femoral epiphysis.
Question 65 Explanation: 
Slipped capital femoral epiphysis is most common in overweight adolescent males who present with complaints of pain that is referred to the thigh or medial side of the knee associated with a limp. X-ray films of the knee are normal since the condition involves the hip.
Question 66
A 23 year-old female presents with ongoing arthralgias with intermittent flares of arthritis. She is found to have a malar rash and an abnormal urinalysis. Serum ANA and anti-double-stranded DNA antibodies are present. Which of the following tests should be ordered to assess her risk for thrombotic events and future risk of spontaneous abortion?
A
complete blood count
Hint:
CBC may reveal anemia, leukopenia and thrombocytopenia, but these are not the cause of thrombotic events and spontaneous abortion in SLE.
B
PT/INR
Hint:
The PT/INR should not be altered in SLE.
C
bleeding time
Hint:
Bleeding time measures platelet function, not risk for thrombosis.
D
anti-phospholipid antibodies
Question 66 Explanation: 
Anti-phospholipid antibodies are present in 25% of SLE patients and may cause thrombotic events and spontaneous abortion.
Question 67
Which of the following fractures is associated with the greatest risk of avascular necrosis of the femoral head?
A
intertrochanteric
Hint:
See answer for explanation.
B
femoral neck
C
subtrochanteric
Hint:
See answer for explanation.
D
pelvic rim
Hint:
See answer for explanation.
Question 67 Explanation: 
Femoral neck fractures lead to the greatest disruption of arterial blood supply to the femoral head.
Question 68
A 22 year-old male presents to the ED after sustaining a blow to the knee during football practice. The knee exam demonstrates significant forward translation of the tibia when the knee is in 20 degrees of flexion and the tibia is forced forward while the femur is stabilized. Which of the following knee maneuvers does this represent?
A
abduction stress test
Hint:
The abduction stress test is performed to evaluate medial collateral ligament tears while applying valgus stress.
B
anterior drawer sign
Hint:
The anterior drawer sign is performed to evaluate the anterior cruciate ligament; however the patient is supine, hips and knees flexed, and feet are flat on the table.
C
Lachman test
D
McMurray test
Hint:
The McMurray test is performed to evaluate medial and lateral meniscal tears while rotating the lower leg internally and externally.
Question 68 Explanation: 
The Lachman test is performed to evaluate the anterior cruciate ligament. The knee is placed in 20 degrees of flexion.
Question 69
Which of the following would demonstrate rotational misalignment in a patient with a fracture of the fourth metacarpal?
A
base of the ring fingernail and index fingernail line up in the partially closed hand
Hint:
Failure of the planes of the fingernails in the partially closed hand to line up indicates rotational misalignment.
B
fingernails of the open hand form an asymmetric arc
Hint:
Fingernails of the open hand normally form an asymmetric arc.
C
ring finger of the closed hand overlaps the little finger
D
ring finger of the open hand is shortened
Hint:
Shortening of a finger does not indicate rotational misalignment.
Question 69 Explanation: 
All fingernails should point to the same spot when the hand is closed. Overlapping of one finger over the other indicates rotational misalignment.
Question 70
A 32 year-old medical transcriptionist presents with burning and tingling in her right wrist and hand for the past month. On physical exam, Phalen's test is positive; however, there is no atrophy of the thenar eminence. Which of the following is the initial step in management of this patient?
A
wrist splints
B
corticosteroid injection
Hint:
Corticosteroid injections and surgery are indicated only after a trial of the wrist splint provides no relief.
C
surgical referral
Hint:
Corticosteroid injections and surgery are indicated only after a trial of the wrist splint provides no relief.
D
propoxyphene (Darvocet)
Hint:
Darvocet has no role in the treatment of carpal tunnel syndrome.
Question 70 Explanation: 
The treatment of carpal tunnel syndrome is aimed at relieving the pressure on the median nerve. This is best accomplished by having the patient wear a wrist splint during the activities that increase the pressure on the median nerve.
Question 71
An x-ray reveals a break in the cortex of one side of the ulna shaft without a separation or break of the opposite cortex describes what type of fracture?
A
greenstick
B
transverse
Hint:
A transverse fracture is a complete fracture of both cortices.
C
torus (buckle)
Hint:
A torus fracture is a bowing, bending, or buckling without a break in the cortex.
D
epiphyseal
Hint:
Epiphyseal fracture occurs at the growth plate.
Question 71 Explanation: 
A greenstick fracture is a break in the cortex of one side of bone shaft without a break in the opposite cortex.
Question 72
A 67 year-old female presents with progressive pain in her left knee that is worse with activity and relieved with rest. She notes stiffness of the knee that last about 20 minutes after activity is resumed. She exercises regularly. No known drug allergies. On physical exam she is 5' 5", 225 pounds. Her left knee exam reveals mild effusion without erythema or warmth. Radiographs of the left knee reveal medial joint space narrowing and subchondral bone sclerosis. Her sodium is 138 mEq/L, potassium 4.3 mEq/L, bicarbonate 24 mEq/L, chloride 104 mEq/L, BUN 23 mg/dl and creatinine 1.8 mg/dl. Which of the following medications is most appropriate for this patient's worsening pain?
A
naprosyn
Hint:
Naprosyn is contraindicated in patients with abnormal renal function.
B
prednisone
Hint:
Oral prednisone is not appropriate for the long-term management of osteoarthritis.
C
acetaminophen
D
methotrexate
Hint:
Methotrexate is not indicated for the treatment of osteoarthritis.
Question 72 Explanation: 
Acetaminophen is recommended as first-line pharmacotherapy in patients with osteoarthritis.
Question 73
An 18 year-old patient has a tibia/fibula fracture following a motorcycle crash. Twelve hours later the patient presents with increased pain despite adequate doses of analgesics and immobilization. Which of the following is the most likely diagnosis?
A
avascular necrosis
Hint:
Avascular necrosis is a late complication of fracture resulting from disruption of the blood supply to the bone.
B
myositis ossificans
Hint:
Myositis ossificans occurs primarily in muscles post-traumatically and may not arise for several months after an injury.
C
compartment syndrome
D
reflex sympathetic dystrophy
Hint:
Reflex sympathetic dystrophy is characterized by painful wasting of muscles that may be secondary to injury and could occur as a late complication
Question 73 Explanation: 
Compartment syndrome is characterized by a pathological increase of pressure within a closed space and results from edema or bleeding within the compartment. It may occur as an early local complication of fracture.
Question 74
A 58 year-old male presents complaining of anterior right shoulder pain the day after performing extensive yard work. The pain is localized over the anterior proximal humerus with distinct point tenderness. There are no visible abnormalities. The patient has full range of motion and strength with all shoulder movements. The pain is reproduced by asking the patient to resist the examiner during supination of the right elbow. Which of the following is the most likely diagnosis?
A
supraspinatus tendonitis
B
subacromial bursitis
Hint:
Supraspinatus tendonitis, subacromial bursitis and rotator cuff tear usually present with pain in the area of the deltoid muscle, limited abduction and are reproduced through impingement tests that narrow the space between the acromium and the humerus thereby impinging the supraspinatus tendon.
C
rotator cuff tear
Hint:
Supraspinatus tendonitis, subacromial bursitis and rotator cuff tear usually present with pain in the area of the deltoid muscle, limited abduction and are reproduced through impingement tests that narrow the space between the acromium and the humerus thereby impinging the supraspinatus tendon.
D
bicipital tendonitis
Question 74 Explanation: 
Bicipital tendonitis presents with anterior shoulder pain that is reproduced by palpating the tendon in the humeral bicipital groove and through resisted motion of the biceps muscle (elbow flexion or supination).
Question 75
The most reliable site from which to identify the causative organism in cases of osteomyelitis is the
A
base of ulcer.
Hint:
Taking specimens for culture from a sinus tract or the base of an ulcer correlate poorly with organisms infecting the bone.
B
blood
Hint:
While blood cultures are indicated in acute cases of osteomyelitis, they are only positive in 25-50% of pediatric hematogenous osteomyelitis and 10% of other forms of bone infection.
C
sinus tract.
Hint:
Taking specimens for culture from a sinus tract or the base of an ulcer correlate poorly with organisms infecting the bone.
D
bone.
Question 75 Explanation: 
Samples from needle aspiration of pus in the bone, or from a bone biopsy, are essential to determine the exact causative agent.
Question 76
Which of the following osteoporosis therapies is so poorly absorbed that it must be taken alone and on an empty stomach?
A
raloxifene (Evista)
Hint:
Raloxifene, calcitonin, and teriparatide lack significant drug-drug or drug-food interactions.
B
alendronate (Fosamax)
C
calcitonin (Miacalcin)
Hint:
Raloxifene, calcitonin, and teriparatide lack significant drug-drug or drug-food interactions.
D
teriparatide (Forteo)
Hint:
Raloxifene, calcitonin, and teriparatide lack significant drug-drug or drug-food interactions.
Question 76 Explanation: 
Fosamax should not be coadministered with any other medication or food.
Question 77
A 43 year-old male with a history of a right medial meniscectomy and a strong family history of osteoarthritis presents to the clinic for a routine physical exam. He states he is very active and runs 20-25 miles a week and competes routinely in 5 km races. He is 5' 10" and 160 lbs, BP is 128/76 and P 72. His physical examination is unremarkable. Which of the following would you recommend to this patient to delay the onset of osteoarthritis?
A
weight loss
Hint:
The patient's BMI is 23. Obesity is not a contributing factor for this patient.
B
corticosteroid injections
Hint:
Corticosteroid injections may be used for symptomatic treatment but are not used for prevention.
C
1500 mg of calcium daily
Hint:
Calcium supplementation is indicated for osteoporosis prevention.
D
consider swimming or biking instead of running
Question 77 Explanation: 
Swimming and biking would promote excellent joint motion and muscle strength and void the high-impact of competitive short-distance running.
Question 78
Which of the following clinical manifestations is commonly seen in a patient with rheumatoid arthritis?
A
symmetric joint swelling
B
presence of Heberden's nodes
Hint:
Heberden's nodes, or bony enlargements of the DIP joints, are seen in osteoarthritis, not rheumatoid arthritis.
C
morning stiffness lasting less than 15 minutes
Hint:
Morning stiffness lasting less than 15 minutes is more characteristic of osteoarthritis. The morning stiffness typically seen with rheumatoid arthritis lasts longer than 1 hour and is a distinguishing feature between the two types of arthritis.
D
cervical spondylosis of C3-C7
Hint:
Cervical spondylosis occurs with osteoarthritis. RA is associated with C1-C2 subluxation.
Question 78 Explanation: 
Symmetric joint swelling associated with stiffness, warmth, tenderness, and pain are characteristic of rheumatoid arthritis.
Question 79
A 20 year-old male presents with pain along the medial tibia. The pain initially began towards the end of soccer practice but now it is present earlier on during practice. Physical exam reveals pain to palpation over the posterior tibialis muscle body. What is the most likely diagnosis?
A
shin splint
B
stress fracture
Hint:
Shin splints cause pain over the posterior tibialis muscle body as opposed to discrete pain over the tibia with a stress fracture.
C
Osgood-Schlatter disease
Hint:
Osgood-Schlatter disease is an injury occurring at the insertion of the patellar tendon on the tibial tuberosity in a younger age group.
D
patellofemoral pain syndrome
Hint:
Patellofemoral pain syndrome is the most common cause of chronic anterior knee pain, more commonly seen in females.
Question 79 Explanation: 
Shin splints cause pain over the posterior tibialis muscle body as opposed to discrete pain over the tibia with a stress fracture.
Question 80
A 36 year-old male has a history of recurrent low back pain. When lifting a stack of books yesterday, he experienced sudden, severe pain in the lumbar area. He denies radicular pain. His lower extremity examination is unremarkable, and his back examination is remarkable for paraspinal muscle tenderness and increased pain with flexion at the waist. Which of the following management options should be instituted at this time?
A
refer the patient to orthopedic surgery
Hint:
The patient can be managed conservatively by a primary care provider since there are no neurological deficits.
B
continue ordinary activities as tolerated
C
confine the patient to bed with traction
Hint:
Traction is an outdated method of treatment, while bed rest for more than a few days will cause muscle atrophy.
D
recommend sleeping on a softer mattress
Hint:
Management for low back strain includes using a firm mattress.
Question 80 Explanation: 
Continuation of activities as tolerated is recommended during the acute phase. After symptoms resolve, an exercise program should be initiated to strengthen the back.
Question 81
A 35 year-old patient with lupus is being treated for mild arthralgias and rash with hydroxychloroquine. Which of the following clinical manifestations is the most common side effect of this medication?
A
impaired night vision
B
jaundice
Hint:
Hydroxychloroquine is not associated with hepatoxicity of hemolytic anemia.
C
proteinuria
Hint:
Hydroxychloroquine lacks renal toxicity.
D
mouth sores
Hint:
Mouth sores may be a sign of agranulocytosis in patients on hydroxychloroquine, but retinopathy occurs more commonly.
Question 81 Explanation: 
Hydroxychloroquine is associated with macular damage, rash and diarrhea.
Question 82
A 42 year-old female experiences pain on the plantar surface of her left foot in the area of the third metatarsal head. The pain is associated with wearing tight shoes and is relieved by removing shoes. Examination reveals a palpable mass and reproduction of pain with deep palpation of the third intermetatarsal space. The patient has tried wearing wider shoes with metatarsal cushions and taking NSAIDS but her symptoms persist. What is the best therapeutic option at this point?
A
Casting of the involved foot
Hint:
Casting the foot in a patient with Morton's neuroma is not effective therapy.
B
Physical therapy
Hint:
Physical therapy has not been shown to be of benefit in treating Morton's neuroma.
C
Steroid injection
D
Surgical excision
Hint:
Surgical excision is recommended for treatment of Morton's neuroma only if conservative measures and steroid injection have failed.
Question 82 Explanation: 
Steroid injection is the treatment of choice for Morton's neuroma when conservative measures fail.
Question 83
A 65-year-old male presents to his primary care physician for stiffness in his arm. He states that he has been having trouble combing his hair and reaching objects that are high on the shelf. The patient has a past medical history of diabetes mellitus type II, obesity, and hypertension. His current medications include metformin, insulin, lisinopril, and hydrochlorothiazide. The patient admits to leading a sedentary life in which he tends to stay home and watch television. He does not engage in any physical or strenuous activity. On physical exam the patient has decreased passive and active range of motion of his shoulder. Strength of the patient's upper extremity is 4/5. Which of the following is the most likely diagnosis?
A
Rotator cuff impingement
Hint:
Rotator cuff impingement typically presents with pain and weakness of the upper extremity but also can present with a decreased range of motion as this patient experienced. Of note, the passive range of motion will not be significantly affected in this disease.
B
Adhesive capsulitis
C
Biceps tendinopathy
Hint:
Biceps tendinopathy presents with pain and weakness overlying the biceps tendon (in the region of the anterior deltoid).
D
Subacromial bursitis
Hint:
Subacromial bursitis presents with localized tenderness, decreased range of motion and pain with motion, and sometimes erythema/edema. Though this patient is experiencing a decreased range of motion and pain with motion, the decreased passive range of motion suggests a diagnosis of adhesive capsulitis.
Question 83 Explanation: 
Adhesive capsulitis classically presents in elderly, sedentary patients with a chief complaint of stiffness. On physical exam, the key findings are a loss of active, but more importantly, passive range of motion. It is the loss of passive range of motion that suggests a diagnosis of adhesive capsulitis over rotator cuff injuries, biceps tendinopathy, glenohumeral osteoarthritis, and subacromial bursitis. The diagnosis of adhesive capsulitis is made clinically, however, radiograph, MRI, and ultrasound can be used to rule out other diagnoses. Treatment includes NSAIDs and physical therapy and can include steroids.
Question 84
A 22 year-old female complains of worsening pain, swelling, and tenderness in her left heel for 1 week. She sustained a penetrating injury to the heel two weeks ago when she stepped on a nail while running in tennis shoes. Examination reveals a draining puncture wound with surrounding erythema and exquisite tenderness. X-ray of the left foot demonstrates periosteal reaction associated with the wound. Which organism is classically responsible for this infection?
A
Escherichia coli
Hint:
While gram negative enteric organisms can be a cause of osteomyelitis, E. coli is rarely seen outside of the neonatal period.
B
Streptococcus pyogenes
Hint:
Streptococcus pyogenes is an uncommon (~10 %) cause of acute osteomyelitis. The mechanism of injury suggests a different pathogen.
C
Pseudomonas aeruginosa
D
Staphylococcus aureus
Hint:
Staphylococcus aureus is the most common infecting organism in cases of acute osteomyelitis, however, the mechanism of injury in this case suggests P. aeruginosa.
Question 84 Explanation: 
Pseudomonas aeruginosa is frequently associated with osteomyelitis involving puncture wounds of the foot. This is believed to result from direct inoculation with P. aeruginosa via the foam padding found in tennis shoes.
Question 85

Which of the following is an established risk factor for osteoporosis?

A
Parity status
Hint:
Parity status and lactation history have been shown to be poor predictors of bone mass; therefore, they do not have an established role in the prediction of increased risk for development of osteoporosis.
B
Carbohydrate intake
Hint:
Carbohydrate intake has no association to the development of osteoporosis.
C
Lactation history
Hint:
Parity status and lactation history have been shown to be poor predictors of bone mass; therefore, they do not have an established role in the prediction of increased risk for development of osteoporosis.
D
Low body weight
Question 85 Explanation: 
Established risk factors for osteoporosis include low body weight, female sex, advanced age, Caucasian race, and bilateral oophorectomy before menopause without estrogen replacement.
Question 86
A 60 year-old female injured her right wrist when she slipped and fell onto her outstretched hand. Radiographs show a fracture through the metaphysis of the distal radius with dorsal displacement and angulation. Which of the following splints is the best method of temporary immobilization?
A
Dorsal forearm
Hint:
The dorsal forearm splint is best used as an alternative to the ulnar or radial gutter splint for protection of fractures of metacarpals two through five.
B
Ulnar gutter
Hint:
Ulnar gutter splints are best for treatment of stable fractures and dislocations of the ulnar wrist and metacarpals.
C
Volar forearm
D
Volar with thumb spica
Hint:
A volar splint with thumb spica is used to immobilize the first metacarpophalangeal joint and is useful for scaphoid fractures.
Question 86 Explanation: 
The volar forearm splint is best for temporary immobilization of forearm, wrist and hand fractures and is the splint of choice for Colles' fracture.
Question 87
An obese 15 year-old male presents with complaint of a limp and right knee pain for two weeks. He denies recent trauma or history of previous injury. Physical examination of the right knee is unremarkable. Examination of the right hip reveals pain with passive range of motion and limited internal rotation and abduction. Flexion of the hip results in external rotation of the thigh. Gait is antalgic with the right hip externally rotated. Which of the following radiographic findings supports the most likely diagnosis?
A
Displacement of the femoral epiphysis
B
Irregularity and fragmentation of the joint space
Hint:
Irregularity and fragmentation of the joint space is associated with avascular necrosis of the femoral head as seen in Legg-Calve-Perthes disease. This typically occurs in a younger male population and is not associated with the classic externally rotated hip with ambulation seen in SCFE.
C
Capsular swelling of the joint
Hint:
Capsular swelling of the joint may be seen in transient synovitis of the hip but is not associated with SCFE.
D
Dislocation of the hip
Hint:
Hip dislocation at this age is associated with major trauma, such as that sustained in a fall from height or dashboard injury. SCFE does not lead to hip dislocation.
Question 87 Explanation: 
This patient has slipped capital femoral epiphysis (SCFE) and the classic x-ray findings will demonstrate displacement of the femoral head rotation of the femoral neck anteriorly.
Question 88
A 57 year-old male presents with acute bilateral lower extremity weakness and urinary incontinence that began after he fell earlier today. His examination is significant for bilateral lower extremity sensory deficits and weakness along with decreased rectal sphincter tone. Which of the following is the most appropriate intervention?
A
Epidural steroids
Hint:
While epidural steroids can be effective in treating lumbar disc herniation, in the case of cauda equina syndrome, immediate surgical decompression is mandatory.
B
Oral NSAIDs
Hint:
NSAIDs may be beneficial in some cases of lumbar muscle strain and disc herniation. They are not appropriate for management of cauda equina syndrome, immediate surgical decompression is mandatory.
C
Physical therapy
Hint:
Physical therapy may be beneficial in some cases of lumbar muscle strain and disc herniation but it is not appropriate for management of cauda equina syndrome.
D
Surgery
Question 88 Explanation: 
Cauda equina syndrome is a rare but serious surgical emergency because the duration of nerve compression is inversely correlated with the likelihood of full neurologic recovery.
Question 89
Which of the following is the most sensitive to determine whether there is a small effusion in the knee?
A
Compress the patella and move it against the femur, noting any crepitus
Hint:
Crepitus without pain is not significant and does not indicate an effusion.
B
Flex the knee to about 90 degrees and palpate for tenderness over the joint line
Hint:
Tenderness over the joint line indicates a meniscal injury, but does not demonstrate an effusion.
C
Milk the medial aspect of the knee, press lateral margin of the patella, and note a bulge of returning fluid medial to the patella
D
Force fluid into space between the patella and the femur, tap the patella over the femur to detect a click
Hint:
Ballottement of the patella against the femur is useful for detecting large effusions, but not small ones.
Question 89 Explanation: 
A small bulge of returning fluid after milking fluid upward from the knee is useful for detecting small effusions
Question 90
A 41 year-old female complains of 3 weeks of gradually worsening pain at the base of the thumb and radial aspect of the wrist. She and her husband have been renovating their home for the past 2 months and it has become increasingly difficult for her to hold a hammer. She denies numbness or tingling. She denies any history of previous trauma to the wrist. On examination, there is tenderness over the distal radial styloid and pain reproduced with ulnar deviation of a fist clenched over the abducted thumb. Which of the following is the most likely diagnosis?
A
Carpal tunnel syndrome
Hint:
Carpal tunnel syndrome typically presents with pain and paresthesias in the median nerve distribution.
B
deQuervain's tenosynovitis
C
Ganglion cyst
Hint:
Ganglion cysts classically present with a visible or palpable, usually painless swelling over the dorsum of the wrist.
D
Volar flexor tenosynovitis
Hint:
With volar flexor tenosynovitis, pain is expected with extension of the fingers and localized tenderness of the volar tendon sheaths.
Question 90 Explanation: 
deQuervain's tenosynovitis typically results from repetitive activity involving pinching the thumb while moving the wrist. There is often pain and tenderness over the radial styloid and Finkelstein's is positive in this patient.
Question 91
A football player complains of burning pain, numbness, and tingling extending from the left shoulder down into the hand after he tackled a player. These symptoms resolved spontaneously in minutes. Following resolution of the symptoms, he has full strength and normal sensation in the left arm. What is the most likely etiology of his symptoms?
A
Acute muscle strain
Hint:
Acute cervical muscle strain might result from the mechanism of injury described but symptoms would not quickly resolve.
B
Mild concussion
Hint:
Concussion involves generalized symptoms such as loss of consciousness but would not affect one limb exclusively.
C
Stretching of nerve roots and brachial plexus y
D
Thoracic outlet obstruction
Hint:
Thoracic outlet syndrome is most commonly caused by cervical rib and is usually only symptomatic when the arm is elevated.
Question 91 Explanation: 
Brachial plexus neurapraxia, commonly called "stinger" injuries, results from stretching of the cervical nerve roots and brachial plexus by a mechanism such as that described in this question.
Question 92

A 43-year-old female presents with a two-year history of frequent episodes of pain and morning stiffness in both hands and wrists. She experiences some symptomatic relief with ibuprofen but feels that the episodes are becoming more frequent and severe. On examination, you observe joint swelling of several MCP joints on both hands. X-ray of the hands shows joint space narrowing of the MCP joints. In addition to NSAIDs, what is the most appropriate first-line long-term medication to treat this patient?

A
Prednisone (Deltasone)
Hint:
Prednisone can produce impressive clinical improvement in rheumatoid arthritis but the side effects associated with their long-term use limit their utility.
B
Infliximab (Enbrel)
Hint:
Tumor necrosis factor inhibitors, such as infliximab, are very effective disease modifying antirheumatic drugs (DMARDS). This class of medication is inappropriate as a first-line medication due to very high cost.
C
Methotrexate (Rheumatrex)
D
Sulfasalazine (Azulfidine)
Hint:
Sulfasalazine is inexpensive but associated with potentially serious hematologic side effects and is considered a second-line DMARD.
Question 92 Explanation: 
Methotrexate is the standard first-line medication in the treatment of rheumatoid arthritis. This DMARD is generally effective, well-tolerated, and affordable.
Question 93
A 13 year-old girl reports two weeks of worsening right knee pain with no history of antecendent injury or recent trauma. She reports frequent episodes of nighttime awakening with knee pain in the past two weeks. Examination of the knee reveals edema and a tender mass over the anterior proximal right tibia. Her knee exam is otherwise within normal limits. Radiographs of the right knee show a lytic mass with a multi-laminated periosteal reaction involving the proximal anterior tibia. What is the most likely diagnosis?
A
Ewing sarcoma
B
Osteochondroma
Hint:
This benign tumor typically presents as a painless mass and appears in plain film radiographs as a stalk or broad-based projection from the surface of the bone.
C
Multiple myeloma
Hint:
The classic radiographic appearance of multiple myeloma is a lytic lesion but this is a condition that is seen in a much older population and is more likely to present with back pain.
D
Osteoid osteoma
Hint:
Although the presentation may be similar to Ewing's sarcoma, the radiographs in osteoid osteoma typically show a round lucency surrounded by sclerotic bone.
Question 93 Explanation: 
The distinctive feature of Ewing sarcoma is the radiographic appearance of a periosteal "onion skin" reaction.
Question 94
A 52-year-old male presents with a severely painful, swollen right great toe. He denies recent trauma but reports several similar episodes of toe pain and swelling over the past two to three years. He has a history of alcohol abuse and hypertension, for which he "takes medication" of an unknown type. Examination reveals bright erythema and edema associated with the right first MCP joint. Which of the patient's antihypertensive medications is most likely contributing to this condition?
A
Diltiazem (Cardizem)
Hint:
Calcium-channel blockers, such as diltiazem, are not associated with increased risk of hyperuricemia and gout.
B
Hydrochlorothiazide (HCTZ)
C
Lisinopril (Prinivil)
Hint:
ACE-I, like lisinopril, can mildly cause hyperuricemia but to a lesser degree than HCTZ
D
Metoprolol (Lopressor)
Hint:
Beta-blockers, such as metoprolol, are not associated with increased risk of hyperuricemia and gout.
Question 94 Explanation: 
Thiazide diuretics, such as hydrochlorothiazide, are associated with increased risk of hyperuricemia and gout.
Question 95
A 22 year-old male presents several hours after sustaining a hand injury when he punched a wall. X-rays of the hand demonstrate fracture of the fifth metacarpal neck with 65 degrees dorsal angulation and a claw hand. What is the most appropriate intervention?
A
Antibiotic treatment and ulnar gutter splint immobilization
Hint:
Antibiotic treatment would be appropriate if the skin was broken and the injury was sustained in a fist fight with the potential for introduced oral flora.
B
Closed reduction and ulnar gutter splint immobilization
Hint:
Reduction followed by splinting is recommended for fifth metatarsal fractures with angulation of 15-40 degrees.
C
Open reduction and ulnar gutter splint immobilization
D
Ulnar gutter splint immobilization only
Hint:
With angulation of greater than 15 degrees, reduction should be performed prior to splinting.
Question 95 Explanation: 
Open reduction is indicated with angulation of greater than 40 degrees.
Question 96
The neuromuscular hallmark of polymyalgia rheumatica is
A
asymmetric muscle pain and stiffness.
Hint:
Polymyalgia rheumatica is associated with pain and stiffness but typically occurs in a symmetric distribution.
B
distal paresthesias.
Hint:
Peripheral neurons are unaffected by polymyalgia rheumatica and is not associated with neuropathic symptoms.
C
muscle weakness.
Hint:
Although patients may appear to have proximal muscle weakness, this is usually related to pain and is not true measurable weakness.
D
proximal muscle pain and stiffness.
Question 96 Explanation: 
Proximal symmetric muscle pain and stiffness, particularly involving the shoulder, neck and pelvic girdle, is the musculoskeletal hallmark of polymyalgia rheumatica.
Question 97
A 74 year-old male presents with one month history of right shoulder pain without any known precipitant. His pain involves an area from the right paraspinous musculature to the right deltoid with occasional radiation down the arm. Pain is worse with movement of the shoulder and is not relieved by acetaminophen. He reports numbness of the right index finger and thumb. Physical examination of the shoulder is limited by pain. There is decreased grip strength and absent pinprick sensation in the index finger and thumb. Relexes are normal. What is the most appropriate initial study to obtain?
A
EMG with nerve conduction
Hint:
Although electromyography and nerve conduction studies would be helpful in localizing the nerves involved, this is not an appropriate first-line diagnostic test.
B
MRI of the right shoulder
Hint:
Although there is pain in the shoulder, this patient presents with cervical radiculopathy and shoulder studies are not appropriate.
C
Radiographs of the neck
D
Radiographs of the right shoulder
Hint:
Although there is pain in the shoulder, this patient presents with cervical radiculopathy and shoulder studies are not appropriate.
Question 97 Explanation: 
Cervical radiculopathy may be due to intervertebral foraminal osteophytes which will be identified on plain film radiographs of the neck making this the appropriate first step in evaluation.
Question 98
A 30 year-old diabetic female complains of persistent numbness in her right thumb and forefinger that has been awakening her from sleep for the past week. She is right hand dominant and denies any history of activities involving repetitive motion of the hands. Which of the following is the next step in the evaluation of this patient?
A
Electromyogram (EMG)
Hint:
EMG may be performed in a patient who is being considered for surgery
B
Nerve conduction velocity (NCT)
Hint:
Nerve conduction test may be performed if operative intervention is being planned.
C
Phalen maneuver
D
Finkelstein test
Hint:
The Finkelstein test is performed when deQuervain's tenosynovitis is suspected.
Question 98 Explanation: 
The Phalen maneuver is used to reproduce the symptoms of carpal tunnel syndrome by flexion of the wrist.
Question 99
A 42 year-old male sustained a closed left tibial fracture in a fall two days ago. He was treated with a cast for immobilization. Acutely, he developed severe pain in his left leg. Examination reveals the anterolateral aspect of the leg to be exquisitely tender to palpation. The patient has extreme pain with plantar flexion. What is the most likely diagnosis?
A
Compartment syndrome
B
Deep venous thrombosis
Hint:
DVT below the knee usually occurs in the posterior compartment and, while potentially painful, should not produce the degree of pain described.
C
Osteomyelitis
Hint:
The pain of osteomyelitis, while potentially severe, should not be greatly exacerbated by manipulation of the overlying muscles.
D
Complex regional pain syndrome
Hint:
Complex regional pain syndrome can present with pain but it is typically a burning pain and often accompanied by vasomotor symptoms.
Question 99 Explanation: 
One of the earliest signs of compartment syndrome is severe pain that occurs with extension of the involved muscles.
Question 100
A 27 year-old male presents with gradually worsening low back pain and stiffness for the past two years. His symptoms are worse upon awakening and gradually improve throughout the day. Lumbosacral flexion is less than 50%. Lumbosacral spine films show erosions in the joint line of both sacroiliac joints. HLA-B27 test is positive. Which of the following is the most appropriate first-line medication for this patient?
A
Indomethacin (Indocin)
B
Methotrexate (Rheumatrex)
Hint:
Methotrexate is beneficial in treatment of rheumatoid arthritis, but there is little evidence that it changes the course of disease in ankylosing spondylitis.
C
Prednisone (Medrol dose pack)
Hint:
Oral prednisone may be used sparingly in ankylosing spondylitis, but is less useful for this condition because of its association with osteoporosis.
D
Sulfasalazine (Azulfidine)
Hint:
Sulfasalazine is potentially useful in the treatment of spondyloarthropathies but studies suggest that it is minimally effective in patients with axial disease.
Question 100 Explanation: 
NSAIDs, such as indomethacin, are the mainstay of therapy in ankylosing spondylitis (AS).
Question 101
A 23 year-old male presents to the emergency department with severe right shoulder pain. He is holding his shoulder in internal rotation. Which of the following would increase your suspicion that he has a posterior glenohumeral dislocation?
A
History of a direct posterior blow to the humerus
Hint:
This is a common mechanism for anterior glenohumeral dislocation.
B
History of seizure
C
Palpable mass in the anterior axilla
Hint:
This is a classic finding associated with anterior dislocation. The mass is the humeral head outside of the glenoid.
D
Palpable space beneath the acromion
Hint:
This is also a classic finding associated with anterior dislocation and the space is where the humeral head should be in a non-dislocated shoulder.
Question 101 Explanation: 
Tonic-clonic seizures can create severe contraction of the internal rotator muscles of the shoulder.
Question 102
A 33 year-old male presents to your office with a complaint of right knee injury associated with pain and swelling. He states he was running after his loose dog and suddenly stopped, hyperextended his knee, heard a pop and noticed immediate swelling. On physical examination, the Lachman test and anterior drawer test demonstrates joint laxity. Which of the following ligaments is most likely injured?
A
Medial collateral
Hint:
Medial collateral ligament injuries often occur with rotational injuries or direct impact to the lateral knee. Tenderness medially with laxity with valgus (medial) stress is noted.
B
Lateral collateral
Hint:
Lateral collateral ligament injury causes pain mostly on the lateral aspect of the knee and patients can experience knee buckling with normal gait. Tenderness laterally with laxity with varus (lateral) stress is noted.
C
Posterior cruciate
Hint:
Posterior cruciate ligament injuries occur with an outside directed force, often a posterior directed force such as a knee striking a dashboard. The patients often do not hear a pop. A posterior drawer test or posterior sag test can be useful in the diagnosis.
D
Anterior cruciate
Question 102 Explanation: 
Anterior cruciate ligament injuries occur with sudden deceleration injuries. Patients often hear a pop and the diagnosis is aided by assessing the anterior drawer test and Lachman test. The immediate swelling as well as laxity with anterior drawer test and Lachman test should raise suspicion of anterior cruciate ligamental injury.
Question 103
A 60 year-old right hand dominant male presents to your office complaining of right shoulder pain and progressively worsening arm weakness. His symptoms are aggravated when working above the shoulder level. On physical examination, there is no evidence of muscle atrophy. He has strong (5/5) adduction of his right shoulder but considerable weakness (1/5) with abduction when raising his arm above his head. Passive range of motion is intact. Which of the following is the most likely diagnosis?
A
Rotator cuff tear
B
Subacromial bursitis
Hint:
Subacromial bursitis causes pain with raising hand above head and has mild degree of weakness.
C
Adhesive capsulitis
Hint:
Adhesive capsulitis (frozen shoulder) causes significant reduction in both active and passive range of motion.
D
Supraspinatus tendonitis
Hint:
Of the four rotator cuff muscles, supraspinatus is most likely to strain causing tendonitis. The symptoms cause limited range of motion due to pain but minimal weakness.
Question 103 Explanation: 
Rotator cuff tears are associated with full passive range of motion but have pain and weakness during active abduction.
Question 104
A 57 year-old male recently on a high protein diet presents with an exquisitely tender, erythematous, warm right great toe. Which of the following is the treatment of choice for this patient?
A
Corticosteroids
Hint:
Corticosteroids are effective in acute gout attacks but are reserved for people with non-steroidal anti- inflammatory agent allergies or contraindications.
B
Colchicine
Hint:
Colchicine is not recommended in the treatment of acute gouty arthritis.
C
Allopurinol
Hint:
Allopurinol is useful in reducing uric acid levels but is not the treatment of choice in acute gouty arthritis.
D
Non-steroidal antiinflammatory agents
Question 104 Explanation: 
NSAID medications are the treatment of choice for acute gouty attacks.
Question 105
A 57 year-old male with history of hypertension presents with acute gouty arthritis. Which of the following anti- hypertensive classes should be avoided in this patient?
A
Thiazide diuretics
B
Beta-blockers
Hint:
See answer for explanation.
C
ACE inhibitors
Hint:
See answer for explanation.
D
Calcium channel blockers
Hint:
See answer for explanation.
Question 105 Explanation: 
Thiazide diuretics are common causes of increased urate levels causing gout attacks.
Question 106
Bony and cartilaginous enlargement of distal interphalangeal joints is commonly seen in which of the following medical conditions?
A
Rheumatoidarthritis
Hint:
Findings in Rheumatoid arthritis include skin nodules, pannus formations and symmetric joint involvement sparing the distal interphalangeal joints.
B
Osteoarthritis
C
Psoriatic arthritis
Hint:
Psoriatic arthritis manifests commonly as sacroiliitis. Asymmetrical arthritis occurs in psoriatic arthritis and causes a sausage like appearance to the fingers. It can also mimic the presentation of rheumatoid arthritis.
D
Gouty arthritis
Hint:
Gout commonly affects the great toe, midfoot, ankle, and knee. Tophi may be present which are subcutaneal deposits of monosodium urate crystals.
Question 106 Explanation: 
Heberden's nodes are commonly seen in primary osteoarthritis.
Question 107
Antinuclear Antigen (ANA) is most commonly associated with and monitors progress of which of the following disorders?
A
Systemic Lupus Erythematosis (SLE)
B
Rheumatoid arthritis (RA)
Hint:
Although present in rheumatoid arthritis, ANA is not specific. A high rheumatoid factor (RF) is more telling for RA.
C
Osteoarthritis
Hint:
Osteoarthritis is not significantly associated with ANA.
D
Ankylosing spondylitis
Hint:
Ankylosing spondylitis is a seronegative disorder and does not reflect RF or ANA levels.
Question 107 Explanation: 
Antinuclear Antigen is the most helpful screening tool for SLE.
Question 108
Which of the following histories is most consistent with rheumatoid arthritis (RA)?
A
Acute onset, morning stiffness, monoarticular joint swelling and tenderness
Hint:
Rheumatoid arthritis has an insidious onset, morning stiffness is typically longer than 30 minutes, and although early RA may have monoarticular presentation, it is typically symmetrical.
B
Insidious onset, morning stiffness, symmetrical joint swelling and tenderness
C
Insidious onset, morning stiffness, monoarticular joint swelling and tenderness
Hint:
Rheumatoid arthritis has an insidious onset, morning stiffness is typically longer than 30 minutes, and although early RA may have monoarticular presentation, it is typically symmetrical.
D
Acute onset, morning stiffness, symmetrical joint swelling and tenderness
Hint:
Rheumatoid arthritis has an insidious onset, morning stiffness is typically longer than 30 minutes, and although early RA may have monoarticular presentation, it is typically symmetrical.
Question 108 Explanation: 
Rheumatoid arthritis has an insidious onset, morning stiffness is typically longer than 30 minutes, and is typically symmetrical in presentation.
Question 109
A 26 year-old male was lifting a heavy object two weeks ago when he felt a sudden onset of low back pain. He describes pain in the low mid back at the belt line aggravated with movement. Radicular symptoms are noted in the left buttock down the leg to the dorsal aspect of the foot. He denies any urine or bowel complaints His examination demonstrates an inability to stand on his toes and a positive straight leg raise. Which of the following is most appropriate diagnostic study in this patient?
A
Computed tomography (CT)
Hint:
See answer for explanation.
B
Magneti resonance imaging (MRI)
C
Discography
Hint:
See answer for explanation.
D
Electromyelogram
Hint:
See answer for explanation.
Question 109 Explanation: 
MRI is the diagnostic study of choice in a patient with suspected disc herniation.
Question 110
A 22 year-old male presents to the emergency department complaining of right hand pain after punching a brick wall. His pain is noted at the ulnar aspect of his hand and worsens with touch and movement. On examination, you notice obvious swelling and tenderness over the dorsum of the right hand proximal to the metacarpal phalangeal (MCP) joint of the fifth digit. Skin is intact. X-ray reveals a fracture of the proximal fifth metacarpal with good alignment. Which of the following is the treatment of choice?
A
Immediate orthopedic consult for surgical fixation
Hint:
Fixation is warranted for displaced fractures, angulated fractures beyond 40 degrees, or extensor lag.
B
Long finger splint extending beyond the MCP
Hint:
A long finger splint would not immobilize the joint above and below and may not offer enough immobilization.
C
Ulnar gutter splint to immobilize fracture site
D
Referral for casting
Hint:
Although you will eventually refer the patient for casting, the choice treatment initiated in the emergency room should be immobilization.
Question 110 Explanation: 
An ulnar gutter splint is the most appropriate care in the emergency room.
Question 111
A 44 year-old female hairdresser presents to the clinic with a two-month history of pain and numbness of her left hand that awakens her from sleep. Your exam reveals electric tingling sensations with percussion over the volar surface of the wrist just proximal to the palmar crease and paresthesias noted within 15 seconds of passively flexing the wrist. Which of the following would be the best initial treatment in this patient?
A
Corticosteroid injection
Hint:
Corticosteroid use is recommended after initial treatment is unsuccessful.
B
Surgical intervention
Hint:
Surgical intervention is reserved for patients not responsive to initial/conservative treatment.
C
Nocturnal splinting
D
Gabapentin (Neurontin)
Hint:
Gabapentin is not indicated in the management of carpal tunnel syndrome.
Question 111 Explanation: 
Splinting the affected wrist is the initial treatment choice and maintains a neutral position of the wrist.
Question 112
A 41 year-old male with a history of intravenous drug abuse presents to your office with acute, nontraumatic right knee pain, chills, and sweats starting 2 days ago. On physical examination, his temperature is 102.9 degrees F. The right knee is erythematous, edematous and tender to palpation and range of motion. Plain knee x-ray reveals soft tissue swelling. Which of the following is most likely the diagnosis?
A
Gouty arthritis
Hint:
See answer for explanation.
B
Septic arthritis
C
Rheumatoid arthritis
Hint:
See answer for explanation.
D
Psoriatic arthritis
Hint:
See answer for explanation.
Question 112 Explanation: 
This patient's signs and symptoms are most consistent with septic arthritis. IV drug abuse places this patient at even greater risk.
Question 113
A 56-year-old white post-menopausal female had a recent surveillance DEXA bone densitometry. The T-score is -2.7 for her right hip. In counseling this patient, in addition to medications, which of the following would you recommend?
A
Weight bearing exercises
B
Water aerobics
Hint:
See answer for explanation.
C
Limit sun exposure
Hint:
See answer for explanation.
D
Daily red wine consumption
Hint:
See answer for explanation.
Question 113 Explanation: 
Weight-bearing exercises are an appropriate adjunct to medication in a patient with osteoporosis. Water aerobics are non-weight bearing. Sun exposure and vitamin D supplementation are useful in preventing and treating osteomalacia.
Question 114
A 62 year-old female presents to the emergency room with significant back pain without radiation after lifting a box weighing approximately 15 pounds. She denies any previous trauma or injuries. Past history includes hysterectomy at age 42 and a 49 pack year smoking history. Her current weight is 107 pounds. Lumbo-sacral spine film indicates a spinal compression fracture at level L4. Which of the following tests would you perform to further assess the patient's findings?
A
Computed tomography (CT) of the spine
Hint:
CT of the spine should be reserved for fractures that remain symptomatic or progress after treatment.
B
Magnetic resonance imaging (MRI) of the spine
Hint:
MRI is a good tool in evaluation of union versus non union fractures and should be reserved for fractures that remain symptomatic or progress after treatment.
C
DEXA scan
D
Technetium-99m bone scan
Hint:
Technetium-99m bone scan is useful in the evaluation of active bone formation (or lack of) and is reserved for fractures that remain symptomatic or progress after treatment.
Question 114 Explanation: 
Low patient weight, smoking, and early estrogen deficiency are all risk factors for osteoporosis. Spontaneous fractures occurring from lifting with above risk factors should be evaluated for bone density.
Question 115
A 34 year-old female construction worker presents with episodic blanching of her fingers when exposed to cold weather. The physical examination of her extremities and digits is normal at this time. Which of the following is the most likely diagnosis?
A
Sjogren syndrome
Hint:
Sjogren syndrome is a secondary cause of Raynauds, but it is not the disorder causing the symptoms.
B
Scleroderma
Hint:
Scleroderma is a secondary cause of Raynauds, but it is not the disorder causing the symptoms
C
Raynaud phenomenon
D
Systemic lupus erythematosus
Hint:
Systemic Lupus Erythematosus is a secondary cause of Raynauds, but it is not the disorder causing the symptoms.
Question 115 Explanation: 
Raynauds phenomenon is caused by vascular spasm when exposed to cold or stressful situations.
Question 116

Which of the following types of hip fracture has the highest risk for avascular necrosis and nonunion?

A
Femoral neck
B
Intertrochanetric
Hint:
Intertrochanteric fractures usually do not cause avascular necrosis and nonunion due to the capsule and blood supply to the femoral head remaining intact.
C
Subtrochanteric
Hint:
Subtrochanteric fractures usually do not cause avascular necrosis and nonunion due to the capsule and blood supply to the femoral head remaining intact.
D
Greater trochanteric
Hint:
Greater trochanteric fractures usually do not cause avascular necrosis and nonunion due to the capsule and blood supply to the femoral head remaining intact.
Question 116 Explanation: 
Fractures involving the femoral neck typically disrupt the blood supply to the femoral head and may cause avascular necrosis and nonunion.
Question 117
Which of the following histories best describes spinal stenosis?
A
Gradual onset of back and thigh pain exacerbated by walking and alleviated by sitting
B
Acute onset of low back pain with radiation of pain to right foot, aggravated by sitting
Hint:
Spinal stenosis is gradual in onset and is improved by sitting.
C
Aching in bilateral buttocks with associated pain felt in shoulder or neck
Hint:
Spinal stenosis, unlike polymyalgia rheumatic, does not have associated shoulder or neck pain.
D
Thigh pain aggravated by walking and absence of pedal pulses
Hint:
True claudication will have thigh pain similar to spinal stenosis but can be differentiated by absence of pulses. Spinal stenosis preserves distal pulses.
Question 117 Explanation: 
Spinal stenosis typically presents as back pain and thigh pain aggravated by ambulation and relieved by sitting.
Question 118
A 13 year-old female presents to the office with right knee and thigh pain and the inability to bear weight since waking yesterday morning. The mother states the child had a fever of 100.9 degrees F this morning and continues to be non- weight bearing. Examination reveals a warm, erythematous, swollen knee. Which of the following tests would be most beneficial in the diagnosis and treatment of this patient?
A
Plain x-ray of the femur
Hint:
Plain x-rays may aid in diagnostic suspicion, usually after a week or two, but do not aid in treatment choice.
B
Antistreptolysin O titer
Hint:
ASO titer is not indicated in a patient with osteomyelitis.
C
Magnetic resonance imaging (MRI) of the femur
Hint:
MRI will detect the early inflammation of osteomyelitis but does not aid in the treatment choice.
D
Culture of joint aspirate
Question 118 Explanation: 
A culture of the joint fluid will confirm the diagnosis and offer information regarding infectious agent.
Question 119
A 14-year old male active in sports, has been complaining of intermittent anterior right knee pain for several months. He denies any specific injuries. On examination, there is no erythema, swelling, deformities, joint laxity, or crepitus. Palpation reveals tenderness over the tibial tubercle and bursa of the right knee. This finding is characteristic of which of the following disorders?
A
Discoid meniscus
Hint:
Discoid meniscus causes clicking over the lateral meniscus during flexion. This disorder is mostly painless and occasionally may cause mild aching or effusion.
B
Osgood-Schlatter disease
C
Chondromalacia patella
Hint:
Chondromalacia patella may demonstrate exaggerated knee valgus and subluxation.
D
Osteochondritis dissecans
Hint:
Osteochondritis dissecans in older children may have effusions, pain, and locking of the joint caused by a portion of the joint surface softening and a shearing leading to a loose fragment.
Question 119 Explanation: 
Osgood-Schlatter disease causes pain at the tibial tubercle and it is caused by fragmentation of the tip of the proximal tibial physis.
Question 120
A patient is 12 hours post-closed reduction of a tibial fracture and is in a long leg cast. Despite pain medication the patient complains of unrelieved pain and cannot move his toes. Which of the following is the most appropriate clinical intervention in this patient?
A
Elevate the leg
Hint:
This measure may be used while the necessary equipment is obtained to bivalve the cast, but it will not treat the underlying condition.
B
Bivalve the cast
C
Encourage ambulation
Hint:
The cast for this patient is too tight and the pressure needs to be released. Ambulation is not appropriate in this patient.
D
Place on PCA pump
Hint:
Increasing the patient's pain medication will not treat the underlying condition.
Question 120 Explanation: 
The cast for this patient is too tight and the pressure needs to be released. Bivalving the cast is the best option for this patient.
Question 121
Which of the following medications is the initial treatment of choice for suppressing the progression of rheumatoid arthritis (RA)?
A
Naproxen sodium (Naprosyn)
Hint:
Antiinflammatory medication may help with the symptoms but does not aid in suppression of RA.
B
Prednisone
Hint:
Corticosteroids are useful in the treatment of symptoms but does not aid in the suppression of disease.
C
Methotrexate (Rheumatrex)
D
Gold salts
Hint:
Gold salts can be used for an acute RA flare, but do not suppress of the progression of the disease.
Question 121 Explanation: 
Methotrexate is the initial treatment choice for RA and aids in suppression of disease.
Question 122
A 30 year-old female presents with left wrist pain after slipping on the ice while walking to her car. On examination, pain is noted on palpation over the anatomical snuff box. X-ray of her wrist shows no identifiable fracture. Which of the following is the most appropriate treatment in this patient?
A
No treatment necessary
Hint:
See answer for explanation.
B
ACE wrap application
Hint:
See answer for explanation.
C
Splint application
D
Immediate orthopedic referral
Hint:
See answer for explanation.
Question 122 Explanation: 
Tenderness in the snuff box should be treated as a suspected scaphoid fracture. The patient should be treated as if it is fractured and placed in a splint with a referral to an orthopedic specialist for further evaluation and repeat imaging.
Question 123
A 25 year-old female with irritable bowel syndrome presents with complaint of upper and lower extremity discomfort. The patient has pinpoint muscle tenderness over the shoulder girdle, low back and hips. Which of the following medications is the most appropriate treatment in this patient?
A
Hydrocodone (Vicodin)
Hint:
Opioid medication is not effective in the treatment of fibromyalgia.
B
Amitriptyline (Elavil)
C
Methotrexate (Rheumatrex)
Hint:
Methotrexate is not effective in treating fibromyalgia.
D
Naproxen sodium (Naprosyn)
Hint:
NSAIDs are generally ineffective in treating fibromyalgia.
Question 123 Explanation: 
Amitriptyline is the treatment of choice for fibromyalgia and is typically administered at bedtime.
Question 124
Which thoracic curvature is an indication for treatment with bracing in an adolescent with scoliosis?
A
Less than 20 degrees
Hint:
20 degrees or less does not normally require treatment
B
20 to 40 degrees
C
40 to 60 degrees
Hint:
40 degrees and greater is resistant to bracing and requires surgical fixation with spinal fusion, which is best done at special centers
D
40 degrees with a lumbar curvature of 30 degrees
Hint:
See B for explanation
E
Greater than 70 degrees
Hint:
A greater than 70-degree curvature is associated with poor respiratory function in adulthood.
Question 124 Explanation: 
Scoliosis is defined by lateral curvature of the spine with rotation of vertebrae and is typically located in the thoracic or lumbar spine in the right or left directions. Idiopathic scoliosis most commonly presents as a right thoracic curve in females from 8 to 10 years of age. Scoliosis is typically asymptomatic unless curvatures are so severe that there is pulmonary dysfunction or there is an underlying disorder (bone or spinal tumor) that is causing the scoliosis. X-rays need to be taken of the entire spine to help determine the degree of curvature. Treatment modalities are based on the degree of curvature: 20 degrees or less does not normally require treatment; 20 to 40 degrees is an indication for bracing in an immature child; and 40 degrees and greater is resistant to bracing and requires surgical fixation with spinal fusion, which is best done at special centers. A greater than 70-degree curvature is associated with poor respiratory function in adulthood.
Question 125
A 28-year-old male is rushed to the emergency department after a late-night altercation in a local bar. During the altercation, he was stabbed with a knife in his left upper extremity just proximal to his antecubital fossa. When he arrives at the emergency department, the knife is still embedded in his distal humerus. What is the most appropriate diagnostic step in ruling out vascular injury in this individual?
A
Check distal pulses by palpation
Hint:
This is not considered the most appropriate method of ruling out vascular injury in an extremity with a penetrating trauma.
B
Check distal pulses by auscultation
Hint:
This is not considered the most appropriate method of ruling out vascular injury in an extremity with a penetrating trauma.
C
Check distal pulses by Doppler ultrasound
D
Check distal pulses by magnetic resonance angiography
Hint:
This is not considered the most appropriate method of ruling out vascular injury in an extremity with a penetrating trauma.
E
Check distal pulses by a combination of palpation and auscultation
Hint:
This is not considered the most appropriate method of ruling out vascular injury in an extremity with a penetrating trauma.
Question 125 Explanation: 
In penetrating wounds to an extremity, the presence of distal pulses does not rule out vascular injury. Perform doppler studies to assess vascular integrity.
Question 126
A 16-year-old female presents to her orthopedic surgeon complaining of foot pain. She states that yesterday, while at cheerleading practice, she was dropped after being thrown in the air. She states that she landed on her right foot and she immediately had pain. Today, she has noticed significant swelling of her right foot behind her ankle. On physical examination, she has excruciating posterolateral pain in her right ankle with plantar flexion and flexion of her great toe. She also has decreased range of motion in her subtalar joint. Ankle radiographs are shown here. What is the most likely diagnosis?
A
Distal tibia fracture
Hint:
Although distal tibia fractures are a result from high energy axial loading mechanisms (motor vehicle accidents, falls from height), the CT scan does not demonstrate this type of injury. Distal tibia fractures often present with severe ankle pain, inability to bear weight, and deformity.
B
Low ankle sprain
Hint:
Low ankle sprains account for 90% of ankle sprains and involve the anterior talofibular ligament and the calacaneofibular ligament.
C
Intra-articular calcaneus fracture
Hint:
Although calcaneus fractures are often seen with traumatic axial loading as the primary mechanism of injury, the radiograph does not show a calcaneus fracture.
D
Posterior lateral talar tubercle fracture
E
Tarsometatarsal fracture-dislocation
Hint:
Tarsometatarsal fracture-dislocations, otherwise known as a Lisfranc injury, is a disruption between the articulation of the medial cuneiform and base of the second metatarsal.
Question 126 Explanation: 
The patient has experienced a posterior lateral talar tubercle fracture. Posterior lateral talar tubercle fractures are seen in athletes who experience axial loading after landing hard following a jump or fall. This fracture is often confused with an ankle sprain and the diagnosis is often very difficult. Active flexion of great toe reproduces pain as the flexor hallucis longus (FHL) moves over the fracture site. If the fracture is displaced, open reduction and internal fixation (ORIF) is the best treatment option. If the fracture is completely non-displaced, then immobilization and non-weightbearing is the preferred treatment.
Once you are finished, click the button below. Any items you have not completed will be marked incorrect. Get Results
There are 126 questions to complete.
List
Return
Shaded items are complete.
12345
678910
1112131415
1617181920
2122232425
2627282930
3132333435
3637383940
4142434445
4647484950
5152535455
5657585960
6162636465
6667686970
7172737475
7677787980
8182838485
8687888990
9192939495
96979899100
101102103104105
106107108109110
111112113114115
116117118119120
121122123124125
126End
Return

St. Patty's Day Sale! Get 20% off your SMARTY PANCE purchase (Two days only)

X